94
Miscellaneous q 51 A 42-year-old female comes to see you in the office for a follow-up visit regarding her hypertension. She has been your patient for the last eight years, during which her blood pressure has been stable on hydrochlorothiazide and amlodipine. She tells you that she recently started taking some "diet supplements" due to the advice of one of her friends. After starting the supplements, her home blood pressure readings became high. She wants your advice on this matter. Her blood pressure in the office today is 152/88 mmHg. Which of the following herbal preparations is associated with hypertension? A. Licorice B. Ginkgo C. Kava D. Black cohosh E. Horse chestnut This subscription is licensed to user: roopika only User ID: 123489 Explanation: The use of complimentary and alternative medicine therapies (e.g., dietary supplements, herbal medicines) has been steadily increasing in the United States for the last several years. A lot of these substances are taken without any specific diagnosis, due to the recommendations of family, friends, and media without prior consultation with the patient's primary care physician. In the absence of any known clinical information on their safety and efficacy, these substances have the potential to cause adverse effects. It is therefore extremely important to ask patients

Miscellaneous 51

  • Upload
    palak32

  • View
    38

  • Download
    0

Embed Size (px)

DESCRIPTION

MIS

Citation preview

Page 1: Miscellaneous 51

Miscellaneous q 51

A 42-year-old female comes to see you in the office for a follow-up visit regarding her hypertension. She has been your patient for the last eight years, during which her blood pressure has been stable on hydrochlorothiazide and amlodipine. She tells you that she recently started taking some "diet supplements" due to the advice of one of her friends. After starting the supplements, her home blood pressure readings became high. She wants your advice on this matter. Her blood pressure in the office today is 152/88 mmHg. Which of the following herbal preparations is associated with hypertension? A. Licorice B. Ginkgo C. Kava D. Black cohosh E. Horse chestnut This subscription is licensed to user: roopika only User ID: 123489 Explanation: The use of complimentary and alternative medicine therapies (e.g., dietary supplements, herbal medicines) has been steadily increasing in the United States for the last several years. A lot of these substances are taken without any specific diagnosis, due to the recommendations of family, friends, and media without prior consultation with the patient's primary care physician. In the absence of any known clinical information on their safety and efficacy, these substances have the potential to cause adverse effects. It is therefore extremely important to ask patients in detail about their current and recent over-the-counter medication use, including the use of alternative remedies. Chronic ingestion of licorice can cause or aggravate hypertension in patients. Licorice inhibits the enzyme 11-beta hydroxysteroid dehydrogenase, thereby preventing the conversion of cortisol to cortisone. The available cortisol binds to mineralocorticoid receptors and causes hypertension, hypokalemia, and metabolic alkalosis. (Choice B) Ginkgo has been marketed as an antioxidant. It is used for the treatment of mild memory loss, dementia, macular degeneration, and peripheral vascular disease. It does not have any significant adverse effects; however, it can interact with aspirin or warfarin, leading to a potential risk of spontaneous bleeding. (Choice C) Kava has been used for the treatment of various disorders. Its concomitant use with alcohol, benzodiazepines, or other prescription sedatives can potentiate their effect and cause excessive drowsiness or disorientation.

Page 2: Miscellaneous 51

(Choice D) Black cohosh has been used for the treatment of premenstrual syndrome and menopausal symptoms. Its excessive use can cause hypotension. It should be used with caution in patients already on antihypertensive medications. (Choice D) Horse chestnut has been used in patients with venous insufficiency or chronic venous stasis. It can inhibit platelet aggregation and can cause bleeding in patients already on aspirin, aspirin-containing products, or warfarin. Educational Objective: The use of licorice can cause or aggravate hypertension in patients, and should not be used in patients with a history of hypertension. Ginkgo does not have any significant adverse effects; however, it can interact with aspirin or warfarin, leading to a potential risk of spontaneous bleeding. 39% of people answered this question correctly; This subscription is licensed to user ID: 123489 only You are the primary care physician for an obese 42-year-old Caucasian man who was diagnosed with diabetes mellitus five years ago. He was recently begun on an insulin regimen that allowed for excellent glycemic control. His medical history is also significant for hypercholesterolemia, which is well managed with atorvastatin. Recently, however, this patient has been missing his follow-up appointments despite frequent reminders from your office staff. After four months elapse, the patient presents to the office. At this visit, his fasting blood glucose is 142 mg/dL and his glycosylated hemoglobin level is 8.1%. Both tests were within normal limits previously. After thorough inquiry, the patient admits that he began to use the herbal preparation gingko biloba. He is proud to use herbal remedies because "it's always better to use natural medicines." Which of the following is a well-known side effect of ginkgo biloba? A. Psychosis B. Hepatotoxicity C. Cardiac arrhythmias D. Stevens-Johnson syndrome E. Bleeding and platelet dysfunction This subscription is licensed to user: roopika only User ID: 123489 Explanation: Gingko biloba leaf extract is an increasingly popular herbal supplement that many patients use as a "memory booster" because of its suggested propensity for increasing cerebral blood flow. Some studies have suggested that gingko is at least somewhat effective in the treatment of intermittent claudication and Alzheimer's disease. Gingko has been associated with a number of side effects, including seizures, headaches, irritability, restlessness, diarrhea, nausea, and vomiting. It is most notorious for increasing the risk of bleeding and its potentiation of the effects of anticoagulant therapy through various mechanisms, including the inhibition of platelet-activating factor (Choice

Page 3: Miscellaneous 51

E). Several cases of serious intracerebral bleeding associated with gingko use have been reported; in most cases, patients were concurrently taking anticoagulant medications. Aconite is an ingredient in some Chinese herbal medicines used to treat pain or heart failure. It is known to cause serious and sometimes fatal arrhythmias (Choice C), including ventricular or supraventricular tachycardia, bidirectional tachycardia, sinus bradycardia with first-degree heart block, bundle branch block with junctional escape rhythm, and torsade de pointes. Hepatotoxicity (Choice B) has been associated with usage of unsaturated pyrrolizidine alkaloids. These compounds are found in several herbal supplements, including comfrey, borage leaf, and coltsfoot. Other herbal supplements known to cause liver toxicity include ephedra, chaparral, germander, and a Chinese medicine called jin bu huan. Psychosis (Choice A) may arise as a side effect of multiple prescription medications. Some examples include corticosteroids, appetite suppressants, quinacrine, antidyskinetics (eg, levodopa, carbidopa, amantadine), and isotretinoin. Stevens-Johnson syndrome (Choice D) may appear on presentation as a more severe form of erythema multiforme. It is characterized by erosion of mucous membranes, small blisters on purpuric macules, and atypical target lesions. Drugs that are known to cause this condition include antibiotics (eg, sulfonamides, aminopenicillins, quinolones, cephalosporins) and anticonvulsants (eg, lamotrignine, phenytoin, carbamazepine). Educational Objective: Gingko biloba leaf extract is used by many patients as a "memory booster" because of its suggested propensity for increasing cerebral blood flow. It is most notorious for an increased risk of bleeding and a potentiation of the effects of anticoagulant therapy through various mechanisms, including the inhibition of platelet-activating factor. 59% of people answered this question correctly; This subscription is licensed to user ID: 123489 only A 79-year-old Caucasian woman is being evaluated at the nursing home because of shortness of breath. She has advanced Alzheimer's disease, hypertension, diabetes mellitus type 2 and chronic obstructive pulmonary disease (COPD). She has lived in the nursing home for the past four years, and requires assistance with all of her basic activities of daily living. She has a daughter and a son. Her son comes to visit her at least three times a week, but her daughter has not visited her during the last two years. Her medications include lisinopril, glipizide, galantamine, and albuterol and ipratropium bromide metered dosed inhalers. Examination shows dry mucus membranes. Heart sounds are tachycardic. Crackles are heard in the right lung base. The abdomen is soft, non-tender and non-distended. Her temperature is 37.8�C (100�F), blood pressure is 140/80 mmHg, pulse is 94/min and respirations are 24/min. Her oxygen saturation is 85% on room air. She is stuporous, and oriented only to person. There is no motor or sensory deficit, but generalized muscular wasting is appreciated. The chest x-ray shows an alveolar infiltrate in the base of the right lung. Her son arrives at the nursing home, and

Page 4: Miscellaneous 51

requests that his mother receive only oxygen therapy. Which of the following is the most appropriate next step in the management of this patient? A. Transfer her to a reference hospital B. Start her on pain medication C. Explain to the son that nothing can be done at this point D. Start her on oxygen therapy E. Start her on antibiotics and oxygen therapy This subscription is licensed to user: roopika only User ID: 123489 Explanation: This patient is deeply demented, completely dependent on others for her care, and is most likely developing pneumonia. Her current condition has made her incapable of making any decisions regarding her health. She needs a surrogate decision-maker to speak on her behalf, and to preserve her right of autonomy. Her son seems to be the most appropriate person for this position, as he is the only direct relative who is constantly visiting her. The physician must always consider the patient's clinical picture and prognosis, and respect the opinion of the surrogate decision-maker. Although it is very unusual, dementia can be an indicator of a terminal condition, and aspiration pneumonia commonly presents towards the last episodes of the disease. If the patient or surrogate feels that no further therapy should be given since the disease is non-reversible, and because the patient's quality of life is poor, the physician has no right to give further treatment. In this case, the physician has to respect the son's request that his mother receive only oxygen therapy. (Choice A) Studies have shown that it is safe to treat elderly patients with pneumonia in a nursing home if they are not critically ill. However, this patient should be transferred to the hospital if her son wants her to be treated with antibiotics and full supportive care. (Choice B) Pain treatment may eventually be needed if the shortness of breath does not improve with oxygen therapy; however, it is not indicated at this point. (Choice C) It is the physician's duty to always act in the patient's best interest. In this case, he may at least alleviate the patient's pain and give comfort, despite the seriousness of the disease. It is unethical to do nothing even when the patient's prognosis or quality of life is poor. (Choice D) Giving oxygen therapy is the most appropriate next step in management because aside from this step being requested by the surrogate, doing so may control the patient's symptoms.

Page 5: Miscellaneous 51

(Choice E) Giving antibiotics to the patient may indicate the physician's inattention and lack of regard for the family (or surrogate's) decision. Educational Objective: Severely demented patients in nursing homes have a poor quality of life and can be viewed as individuals who are in the terminal phase of a prolonged and debilitating illness. The physician has no right to give or withhold potential life-saving measures based on only his own values, personal evaluation, or opinion of what the acceptable quality of life is. Decisions regarding the treatment of acute and life-threatening conditions have to be made with the surrogate, who acts to preserve the patient's right to autonomy. On the other hand, the physician cannot be forced to provide unnecessary therapy only because the surrogate of relatives request so. 46% of people answered this question correctly; This subscription is licensed to user ID: 123489 only An 18-year-old woman who recently immigrated to the United States from Africa presents to her primary care physician requesting a "female circumcision." She says that this matter is very important to her because it is considered a rite of passage in her culture. Her family feels strongly that she should undergo the procedure in preparation for marriage. Two of her older sisters have already undergone the procedure. Her medical history is unremarkable and physical examination reveals no abnormalities. Which of the following is the most appropriate response? A. "Female circumcision is not allowed in this hospital." B. "I recommend you undergo female circumcision in Africa or the Middle East as they are more experienced with this custom." C. "Female circumcision can have an adverse effect on your health and is a painful and irreversible procedure." D. "Let me refer you to the gynecology department." E. "Let me refer you to the surgery department." This subscription is licensed to user: roopika only User ID: 123489 Explanation: Practiced for centuries on young girls throughout much of Africa, female circumcision is the partial or total cutting away of the external female genitalia in preparation for womanhood and marriage. The procedure is often performed in an unclean setting without anesthetic by those with little understanding of human anatomy or fundamental medical principles. Potentially fatal complications include infection, hemorrhage, and shock. Commonly, women who have undergone female circumcision will go on to develop chronic pain, abscesses, recurrent urinary tract and reproductive tract infections, or infertility. Therefore, this woman who is inquiring about female circumcision should be advised that the procedure can have an adverse effect on her health and that it is a painful and irreversible procedure.

Page 6: Miscellaneous 51

(Choice A) Medically unnecessary surgery on the genitalia of girls younger than 18 years was banned by the United States Congress in 1997. While the requested procedure is technically legal for adult women, few physicians in the United States are comfortable performing it. Ultimately, educating the woman about the health risks associated with the procedure is more likely to be effective in discouraging her than is informing her of its unavailability. (Choice B) Recommending that this patient travel to another region of the world to undergo a procedure that poses serious risk to her health and well-being is not appropriate. (Choices D and E) Referrals to the gynecology or surgery departments would not be appropriate as this procedure is generally not performed in the United States. Educational Objective: Women who are interested in undergoing female circumcision should be advised of the serious health risks associated with the procedure. 75% of people answered this question correctly; This subscription is licensed to user ID: 123489 only A 39-year-old disheveled homeless woman walks into the emergency department complaining of abdominal pain. The woman smells strongly of alcohol and seems disoriented. She describes the abdominal pain as "real bad" but cannot specify when the pain first started, where it is localized, or what factors exacerbate or relieve the pain. She then adds that she does not have health insurance and has no money to pay for her care. What is the next best step in handling this situation? A. Inform patient that she cannot receive care unless she pays in advance B. Ask patient's family members to either assume liability for her bill or take her home C. Provide appropriate medical screening exam and stabilize her condition D. Transfer patient to the county hospital by ambulance immediately E. Refer patient to a local free clinic for follow up next week This subscription is licensed to user: roopika only User ID: 123489 Explanation: Congress enacted The Emergency Medical Treatment and Active Labor Act (EMTALA) in 1996. This law was designed to prevent hospitals from inappropriately transferring, discharging, or refusing to treat indigent patients, and allows for strict fines to be levied on any hospital found in violation of the Act. EMTALA imposes three primary requirements on hospitals that provide emergency services. First, the hospital must provide an appropriate screening medical exam to anyone who comes to the ED seeking medical care (Choice C). Second, if such an individual has an emergency medical

Page 7: Miscellaneous 51

condition, the hospital must treat and stabilize the emergency condition. Third, the hospital must not transfer an individual with an emergency medical condition that has not been stabilized unless several complex conditions are met. Refusing care without advance payment from the patient (Choice A) or from her family members (Choice B) is in violation of EMTALA and therefore incorrect. An appropriate medical screening exam and stabilization should be done before transferring the patient to any other facility (Choice D). Moreover, any such transfers must be arranged in compliance with EMTALA. Referring the patient to a free clinic (Choice E) is also in violation of EMTALA. It is possible that this patient is quite ill and in need of urgent medical care. To postpone evaluation until next week could endanger her life. Educational Objective: The Emergency Medical Treatment and Active Labor Act (EMTALA) was enacted by Congress to prevent hospitals from inappropriately transferring, discharging, or refusing to treat indigent patients. All patients who present to the emergency department must receive an appropriate screening medical exam and stabilization of their condition, regardless of ability to pay. 97% of people answered this question correctly; This subscription is licensed to user ID: 123489 only The following vignette applies to the next 2 items A 41-year-old Caucasian man is brought to the emergency department by ambulance after being in a motor vehicle accident. He was driving a car with his 6-year-old son in the backseat when they were broadsided by a large truck traveling at high speed. He was initially unconscious at the scene of the accident but awoke while in transit to the emergency department. He informed the paramedics that he is a Jehovah's Witness adherent and does not want blood transfusions for himself or his son. He is moaning and writhing about, clearly in distress. He complains of feeling nauseated and thirsty. A facemask is placed to administer 100% oxygen. Initial assessment indicates his temperature is 37C (98.6F), blood pressure is 100/58 mm Hg, pulse is 132/min and thready, and respirations are 34/min. There are numerous lacerations apparent on his scalp, trunk, abdomen, and extremities. His skin is cool and clammy. Chest auscultation reveals tachypnea and tachycardia but is otherwise normal. His abdomen is diffusely tender to palpation and guarding is present. Rectal examination is normal. His blood pressure begins to drop rapidly and approaches 50/0. The trauma surgeon advises that an immediate exploratory laparotomy is warranted to ascertain the source of the internal bleeding. His Hb is 5.5 g/dL. Item 1 of 2

Page 8: Miscellaneous 51

Given the circumstances, which of the following actions would be most appropriate to undertake next? A. Halt further intervention because of patient's religious affiliation B. Administer intravenous fluids and vasopressors C. Ask patient's wife to authorize blood transfusion for her husband D. Petition court to authorize blood transfusion for patient E. Proceed with immediate blood transfusion This subscription is licensed to user: roopika only User ID: 123489 Explanation: It is important that medical personnel honor the principle of patient autonomy when competent adult Jehovah's Witness adherents indicate they do not want to receive blood transfusions. However, other supportive measures should be implemented whenever possible to maximize the likelihood of a successful outcome. For instance, some surgeries can be successfully performed using blood-sparing techniques or erythropoietin. Similarly, in cases of blood loss, intravenous fluids and vasopressors should be used (Choice B). Halting all further intervention (Choice A) would be premature, as the administration of intravenous fluids and vasopressors may stabilize a hypotensive patient. Since the patient is a competent adult, it is inappropriate to seek the permission of either his wife or the courts for a blood transfusion (Choices C and D). The patient has already made his wishes clear. Ordering a blood transfusion (Choice E) in this instance would be in direct violation of the patient's wishes and therefore invites litigation. However, had the patient been unconscious (and his desire to avoid blood products unknown), medical personnel would not have been held legally liable for proceeding with an immediate transfusion. Educational Objective: Intravenous fluids and vasopressors are important supportive measures that should be administered to Jehovah's Witness adherents who have suffered significant blood loss. 90% of people answered this question correctly; This subscription is licensed to user ID: 123489 only Item 2 of 2 Also severely injured in the motor vehicle accident was the man's 6-year-old son. The boy suffered a significant crush injury to his chest and is now unconscious and bleeding profusely. At initial assessment, his temperature was 37C (98.6F), blood pressure was 72/38 mm Hg, pulse was 154/min, and respirations were 42/min. While his chest wound

Page 9: Miscellaneous 51

is externally examined, his blood pressure begins to decline precipitously. Two large-bore intravenous catheters are inserted and fluids are aggressively administered, but the child becomes increasingly hemodynamically unstable. His Hb is 6 g/dL. Patient is intubated. Chest tubes are being placed. What is the most appropriate next step? A. Proceed with immediate blood transfusion B. Call patient's mother to authorize blood transfusion for her son C. Petition court to authorize blood transfusion for patient D. Consult with hospital ethics committee for guidance E. Continue intravenous fluids and do not give blood transfusion This subscription is licensed to user: roopika only User ID: 123489 Explanation: In providing medical care, clinicians must seek to balance the autonomy of the family with the welfare of the child. In urgent situations such as this one, clinicians must act to preserve the child's life, even if such actions contradict the parental wishes. Therefore it is best to proceed with immediate blood transfusion (Choice A). Obtaining approval from either the child's mother (Choice B) or the court (Choice C) requires the luxury of time. Since this is an urgent situation, it is more important to act to save the child immediately. In instances where the child requires some sort of intervention within the next few days but is currently stable, it would be appropriate to seek approval from the courts to proceed with medically necessary treatments. Consulting with the hospital ethics committee (Choice D) should be done when a clinician is confronted with an ethically difficult but non-emergent situation. Halting further intervention (Choice E) would be an acceptable choice if the patient were a competent adult. However, since this patient is a child and considered unable to fully understand the consequences of making such a choice, it is standard practice to act to preserve the child's life in spite of the parental wishes. Educational Objective: Clinicians must act to preserve the life of a child, even if such actions are in direct contradiction with parental wishes. A prime example of this is the standard practice of administering emergency blood transfusions to the children of Jehovah's Witness adherents. 72% of people answered this question correctly; This subscription is licensed to user ID: 123489 only An 82-year-old Caucasian man is admitted to the hospital after an out-of-hospital cardiac arrest. He was successfully resuscitated and intubated in the field by paramedics, and transferred to your intensive care unit. On day three of hospitalization, he remains unresponsive to all stimuli, and is deemed braindead by the criteria. His wife brings in the

Page 10: Miscellaneous 51

advance directives that day, which clearly states that the patient does not wish to be on any kind of life sustaining treatment, including mechanical ventilation. The wife agrees with the advance directive, and wants to proceed with withdrawal of life support. The rest of the family, including three sons and two daughters, gets very upset with the decision, and strongly oppose the withdrawal of life support. They want to have "at least two weeks" before the final decision regarding withdrawing mechanical ventilation can be made. Which of the following is the most appropriate next step in the care of this patient? A. Terminate all life sustaining support now. B. Terminate all life sustaining support after 2 weeks if there is no improvement in the patient's condition. C. Talk with the family and discuss any secondary gains that may be affecting their decision to withhold life support. D. Bring up the issue with the ethics committee of the hospital. E. Move a petition in the court against the family's decision. This subscription is licensed to user: roopika only User ID: 123489 Explanation: Physicians have an obligation to relieve pain and suffering, and to promote the dignity and autonomy of dying patients in their care. If the performance of these duties conflict with the interests of other people involved in the patient care, the preferences of the patient should prevail. The principle of patient autonomy requires that the physicians should respect the decision to withhold or withdraw life-sustaining treatment of a patient who possesses decision-making capacity. Life-sustaining treatment is any treatment of a patient that serves to prolong his life without reversing the underlying medical condition. A competent adult patient may, via an advance directive, formulate and provide valid consent to withhold or withdraw life-supporting treatment in the event that any injury or illness renders that individual incompetent to make such a decision. If the patient was previously clear in expressing his wishes, and is subsequently rendered incapable of making a decision, the physician has the obligation to respect these wishes, and inform the family/surrogate about the patient's prior statements. (Choice B) The decision to withdraw life support should be made now, and not after two weeks. (Choice C) There is no evidence of any secondary gains that could affect the decision to withhold life support. Furthermore, the patient has previously stated his wishes in an advance directive. Bringing up this issue would be inappropriate. (Choices D and E) The use of courts or an ethics committee to resolve a dispute and aid in sound decision making is necessary only in incompetent patients who do not have an advance directive or a surrogate decision maker. Educational Objective:

Page 11: Miscellaneous 51

Patient preference should always prevail in case of any conflict regarding the withdrawal of life support. 67% of people answered this question correctly; This subscription is licensed to user ID: 123489 only A 40-year-old woman with multiple medical complaints comes to your office for an evaluation. She has rheumatoid arthritis, peripheral venous insufficiency, asthma, obesity and depression. She has been evaluated by multiple physicians, and has had multiple investigations. She tells you that her previous doctor asked her every time to undress so that he could examine her breasts. From the hospital administration, you find out that the same physician had prior sexual harassment cases filed against him. What is the best way to approach this situation? A. Ask the patient to file a lawsuit B. Ask the patient to write a letter to the hospital administration C. Notify the hospital administration of the physician's alleged behavior D. Notify the state medical board of the physician's alleged behavior E. Inform the physician about the patient's complaint This subscription is licensed to user: roopika only User ID: 123489 Explanation: The American Medical Association (AMA) recommends all physicians to denounce any colleague whose conduct is unethical, incompetent, corrupt or dishonest. The Office of Physician's Medical Conduct (OPMC) is a part of the state medical board that is in charge of investigating such cases. Physicians are obliged to report to their local OPMC when they have any information about unprofessional behavior, either if a patient informs them about a case of misconduct, or if the physician directly observed the act itself. In this case, the physician has allegedly performed unnecessary breast exams, and should be reported to the OPMC or medical board for further investigation. (Choices A and B) It is not appropriate to advise the patient to file a lawsuit or write a letter of complaint to the hospital administration, unless the patient insists that she has been damaged (either physically or emotionally) due to the acts of misconduct, and wants compensation. (Choice C) The physician has allegedly engaged in unethical conduct, which could lead to the suspension or termination of his license. The hospital administration has no role in this matter, unless the patient complains directly to them. (Choice E) It is legally not advisable for a physician to warn his/her colleague about a patient's concern or accusation of misconduct, because if the colleague is found guilty after the investigation, the physician who warned him can be considered as an accomplice, and can be sued as well. For this reason, as well as for the improvement of

Page 12: Miscellaneous 51

the current standards of medical practice, the AMA advises all physicians to be denouncers against such colleagues. Educational Objective: A physician must inform the state medical board of any colleague who is being suspected or accused of unethical conduct in order to start a thorough investigation. Ideas against or in favor of the accused physician must not be given to the patient. 26% of people answered this question correctly; This subscription is licensed to user ID: 123489 only A 39-year-old Caucasian female attorney calls her primary care physician to ask if she can periodically send him inquiries via email about her health care concerns. She says that she prefers to use email to avoid the hassles of waiting at the doctor's office and scheduling appointments by telephone. She has a medical history of sarcoidosis, gastroesophageal reflux disease, and tension headaches. Her conditions have been managed by the primary care physician for the past five years, and her current medications include omeprazole and ibuprofen. She is reliable in keeping her appointments. Which of the following is the most appropriate response to her request? A. "I'm sorry, but all health care concerns must be addressed at face-to-face visits." B. "We can proceed in this manner if you wish, but be aware that any email correspondence will not be considered confidential." C. "I will honor your request, and will print out all of our email correspondence to include in your medical record." D. "I will have to consult with the hospital administration as exchanging email with patients is not standard practice." E. "Email correspondence crosses professional boundaries and undermines the doctor-patient relationship." This subscription is licensed to user: roopika only User ID: 123489 Explanation: According to the American Medical Association, "Patient-physician electronic mail is defined as computer-based communication between physicians and patients within a professional relationship, in which the physician has taken on an explicit measure of responsibility for the patient's care." Efficient and convenient, email provides the opportunity for follow-up care and clarification of medical advice given in the office setting. Suggested educational resources and websites can be easily referred to in the context of an email, allowing patients to better understand pertinent health care issues. Moreover, email correspondence is easily documented by printing out the patient inquiry and physician response and inserting both into the medical chart. The physician in this scenario should inform the patient that her request will be honored and that all email correspondence will be included in her medical record.

Page 13: Miscellaneous 51

(Choice A) There is no reason that simple health care issues (eg, medication refills, questions about pre- or post-operative care, appointment scheduling, advice clarification) cannot be addressed via email communication. However, patients should be advised that urgent medical issues should be addressed in person or at least by telephone to ensure the promptness of a response. (Choice B) Efforts should be made to maintain confidentiality of all email correspondence, and patients should be asked in advance if they want sensitive matters (eg, mental health) to be discussed via email. Encryption may be advisable to maximize the security of email correspondence. (Choice D) There is no need to consult with hospital administration about email correspondence between doctors and patients, as it is becoming increasingly standard practice. (Choice E) It is certainly possible for a physician to behave in an unprofessional manner via email, just as it is possible in person or by telephone. When used appropriately, however, this form of communication is actually more likely to enhance the doctor-patient relationship, improve rapport, and increase patient satisfaction. Educational Objective: Patient-physician email communication regarding simple, non-urgent health care issues is useful in clarifying instructions, improving rapport, and increasing patient satisfaction. Such correspondence should be printed out and included in the medical chart. 35% of people answered this question correctly; This subscription is licensed to user ID: 123489 only The following vignette applies to the next 2 items A 17-year-old Caucasian male is brought to the emergency department by his friend due to acute shortness of breath. He is apparently in acute distress and speaks in broken sentences. His friend says that they were playing soccer and he went to the bushes to retrieve the ball. Soon after, he returned to the field and complained of shortness of breath and itching. The patient does not recall any bite or injury. The physical examination reveals widespread wheals all over the body. Item 1 of 2 What is the best next step in the management of this patient? A. Complete physical examination looking for a tick B. Epinephrine subcutaneously C. Epinephrine IV followed by IV fluids D. IV hydrocortisone and fluids E. IV diphenhydramine and furosemide

Page 14: Miscellaneous 51

This subscription is licensed to user: roopika only User ID: 123489 Explanation: The clinical scenario described is consistent with an anaphylactic reaction. The two most typical causes of death in patients with anaphylactic reaction are respiratory failure and cardiovascular collapse. Respiratory failure usually results from airway obstruction due to bronchospasm and/or laryngeal edema; other complications that lead to respiratory failure are cardiogenic or noncardiogenic pulmonary edema, or acute respiratory distress syndrome. Cardiovascular collapse results from increased vascular permeability and hypovolemia, alterations in peripheral vascular resistance, and myocardial depression. Due to the potentially life-threatening nature of anaphylaxis, prompt treatment is required. Epinephrine is the drug of choice for anaphylaxis because it can reverse associated hypotension and bronchospasm. This patient seems to be in acute distress (broken speech indicates high degree of respiratory compromise) and should be given epinephrine IV. In addition, fluid support should also be given to compensate for the relative hypovolemia due to the increased vascular permeability. (Choice A) Epinephrine should be given immediately because fatality rates are highest in patients whose treatment with epinephrine is delayed. (Choice B) Subcutaneous epinephrine can be used in patients without significant respiratory or cardiovascular symptoms. (Choices E and D) Antihistamine drugs and corticosteroids are usually given to patients with anaphylactic reactions, but these are less important in reversing acute life-threatening complications than epinephrine. Educational Objective: Epinephrine is the drug of choice for anaphylaxis because it can reverse the associated hypotension and bronchospasm. It should be given immediately because fatality rates are highest in patients whose treatment with epinephrine is delayed. 40% of people answered this question correctly; This subscription is licensed to user ID: 123489 only Item 2 of 2 The patient is appropriately treated. He states that he has never had such symptoms before, which scared him, but he wants to go home now. The most recent physical examination reveals minimal rash and clear lungs on auscultation. What is the most important step in the management of this patient? A. Immediate discharge B. Advise to wear a sign that indicates his allergy C. Prescribe a daily antihistamine drug

Page 15: Miscellaneous 51

D. Instruct how to use EpiPen injector E. Advice not to play on that playground This subscription is licensed to user: roopika only User ID: 123489 Explanation: The most important intervention from the list is instructing the patient about the proper use of EpiPen. EpiPen is an epinephrine automatic injector which should always be immediately available in case of recurrence. Such a device may be lifesaving if patients are inadvertently re-exposed to the offending agent. The EpiPen for adults delivers 0.3 mL of 1:1000 epinephrine (0.3 mg). The injector is administered by taking off the cap and pushing the opposite end firmly into the upper lateral thigh. The needle is delivered into the thigh automatically. The patient should count to 10 before removing the pen to insure complete delivery of the medication. The patient should also be instructed to obtain immediate medical care after the injection of the drug. (Choice B) Another important measure would be wearing a Medic Alert bracelet or similar device at all times. This information can expedite diagnosis and appropriate treatment. (Choice A) Because of the potential for biphasic reactions, the patient should be observed for as long as 24 hours in severe episodes of anaphylaxis. (Choice E) The patients should be instructed to avoid the exposure if the allergen that caused the reaction is identified. (Choice C) Daily antihistamines are prescribed to patients with frequent anaphylactic reactions if the allergen can not be determined or avoided. Educational Objective: The most important intervention following an episode of anaphylaxis is instructing the patient about the proper use of EpiPen. 82% of people answered this question correctly; This subscription is licensed to user ID: 123489 only You are managing the care of a 32-year-old football player in the intensive care unit. He was involved in a motor vehicle accident two days ago, and had sustained severe head injuries. The paramedics intubated him at the scene of the accident. On the third day of hospitalization, he remains unresponsive to all stimuli and has no spontaneous respiratory drive. He is on full ventilator support. His temperature is 36.7 C (98 F), blood pressure is 120/70 mmHg, and pulse is 76/min. He is declared braindead by the ICU team. His family is informed of his status, and they wish to proceed with the withdrawal of life support. His fiance informs you that he had always wanted to donate his organs in case of an unexpected death. Which of the following is important to maximize the viability of donor organs?

Page 16: Miscellaneous 51

A. Anti-hypertensive medications to keep systolic blood pressure less than 100 mmHg B. Low-dose beta blockers to keep heart rate less than 60/minute C. Lower the ventilator rate to induce hypercapnia D. Maintain body temperature to less than 35 C (95 F) E. Maintain normothermia with blankets This subscription is licensed to user: roopika only User ID: 123489 Explanation: Organ transplantation from cadaveric donors has become increasingly common in the last few years. Most of the organs are obtained from brain dead donors. The successful recovery of viable organs for transplantation depends on the appropriate medical care of brain dead patients; therefore, it is important to learn the basic principles of management of these patients. The primary aim is to achieve hemodynamic stability and to maintain physiologic homeostasis to maximize the viability of organs. A common misconception is that the body temperature should be kept low to improve the viability of organs. In contrast, normothermia should be maintained passively in all brain dead patients using blankets. If the body temperature is less than 35 C (95 F), active rewarming should be attempted with warm air blankets and warm intravenous fluids. (Choices A and B) It is critical to achieve and maintain a normotensive, euvolemic state. Hypotension is very common in brain dead patients, and may be due to the loss of sympathetic tone, systemic infections, volume depletion secondary to losses, and diabetes insipidus. Adequate volume resuscitation is the first step in correcting hypotension and ensuring donor viability. If systemic blood pressure still remains low, pressors and inotropic agents may be used to improve the blood pressure. (Choice C) Hypercapnia has not been shown to improve organ viability, and can be even damaging to the tissues and organs. The goal of management is to provide optimal ventilator support to prevent hypoxia and hypercapnia. Educational Objective: The primary goal of medical management of a brain dead organ donor is to maintain a euvolemic, normotensive, and normothermic state. 49% of people answered this question correctly; This subscription is licensed to user ID: 123489 only An 83-year-old Caucasian male with a history of advanced colon carcinoma with metastasis to the lungs develops right-sided hemiplegia and dysarthria while at home. His granddaughter calls you at 8:00 p.m., giving you a history compatible with a cerebrovascular event, probably an ischemic stroke. You know this patient from before. He is not a hospice patient, and he has always firmly expressed his decision to not be

Page 17: Miscellaneous 51

resuscitated. He has a current, valid DNR (do not resuscitate) order. Which of the following is the most appropriate next step in the management of this patient? A. Give him pain medication and aspirin and follow-up tomorrow in the office. B. Admit him to the hospital. C. Explain to her that you will arrange for hospice early in the morning. D. Tell her the patient's prognosis is poor and leave him at home. This subscription is licensed to user: roopika only User ID: 123489 Explanation: A stroke is a serious condition that requires admission to the hospital for evaluation, stabilization, and brain imaging studies. A DNR order implies the commitment of the physician to not attempt resuscitation if the patient's condition seriously deteriorates (e.g. cardiac or respiratory arrest), and does not imply "do not treat." (Choices C & D) Management at home or arranging for hospice is incorrect. The patient deserves to be treated for his concurrent or new conditions according to standard practice. (Choice A) Pain medication may be needed, but it is not an urgent measure to take, especially if the chief complaint of the patient does not involve pain at this point. Educational Objective: A DNR order implies "Do Not Resuscitate." It does not mean, "Do not treat." If the cancer patient is clinically competent and does not wish to be admitted to the hospital, he has the right to decide to stay home. Otherwise, he deserves to be treated for his concurrent or new conditions according to standard practice. 87% of people answered this question correctly; This subscription is licensed to user ID: 123489 only A mother brings her 4-year-old son to the emergency department. She cannot speak or understand English. She can speak only Spanish. Neither you, nor any of your emergency room staff, knows Spanish. Which of the following is the most appropriate choice to communicate with this patient? A. Send them home until the translator is available. B. Ask the mom to bring a translator. C. Try to communicate with the child. D. Call the long distance company for a translator. E. Do the physical examination of the child and try to determine the reason for his visit. This subscription is licensed to user: roopika only User ID: 123489 Explanation:

Page 18: Miscellaneous 51

Almost all the hospitals in the United States have translators available. As a backup, hospitals also obtain accounts with long distance companies who provide translation services. The doctor uses a speakerphone, and a long distance company translator, on the other end of the phone line, assists in obtaining an adequate medical history and physical examination from the patient. (Choices A and B) Sending the patient home or requesting the mother to bring a translator is inappropriate. Doing so may waste a lot of time, and can result to unnecessary delays in treatment. (Choice D) The child is too young to reliably communicate with, and it is unlikely that the child will be conversant with English at this age, especially since his mother speaks only Spanish. (Choice E) Examining the child to determine the problem without first obtaining a medical history is an incorrect approach to this patient's management. Educational Objective: Almost all the hospitals in the United States have translators available. As a backup, hospitals also obtain accounts with long distance companies who provide translation services. 55% of people answered this question correctly; This subscription is licensed to user ID: 123489 only A 53-year-old man is admitted to the hospital after presenting to the emergency department complaining of fever, chills, abdominal pain and swelling, and diarrhea. Bacterial peritonitis is diagnosed, though no specific etiology is yet identified. His history is significant for epilepsy, depression, eczema, and a remote appendectomy. He was diagnosed with epilepsy ten years ago after having three seizures in close succession and has not experienced any seizures since being placed on phenytoin. He uses no other medications at this time. He no longer drinks alcohol and has never smoked tobacco or use recreational drugs. Despite continuous intravenous fluid hydration, his blood pressure begins to decline. Within hours of admission, his blood pressure reaches 88/64 mm Hg, his pulse rises to 132/min, and he is transferred to the intensive care unit. Blood and peritoneal fluid cultures are pending. Laboratory evaluation reveals the following: Serum chemistrySodium 144 mEq/L Potassium 3.7 mEq/L BUN 22 mg/dL Creatinine 1.0 mg/dL CBCHb 15.8 g/dL MCV 96 fl

Page 19: Miscellaneous 51

Platelet count 367,000/cmm Leukocyte count 23,400/cmm Given the circumstances, which antibiotic is contraindicated in the treatment of this patient? A. Doxycycline B. Gentamicin C. Imipenem D. Tobramycin E. Vancomycin This subscription is licensed to user: roopika only User ID: 123489 Explanation: Most antibiotics are not neurotoxic, but some are associated with an increased risk of seizures. Conditions that would predispose an individual to antibiotic-induced seizure include renal insufficiency, older age, pre-existing CNS disease, and concomitant use of proconvulsant drugs. Of all antibiotics, beta-lactams are the most commonly associated with adverse CNS events. Specifically, penicillins, cephalosporins, monobactams, carbapenems (including imipenem), and fluoroquinolones are the antibiotics most likely to trigger seizures (Choice C). Doxycycline (Choice A) is a tetracycline derivative used to treat conditions such as Lyme disease or chlamydial infection. Concerning side effects include bone discoloration and photosensitivity. There is no known increased risk of seizure. Gentamicin (Choice B) is an aminoglycoside often used in the treatment of systemic infections. Concerning central nervous system side effects include vertigo, ataxia, and ototoxicity. There is no known increased risk of seizure. Tobramycin (Choice D) is an aminoglycoside often used in the treatment of systemic infections and cystic fibrosis. Central nervous system side effects are usually mild and can include confusion, lethargy, dizziness, headache, and vertigo. There is no known increased risk of seizure. Vancomycin (Choice E) is an antibiotic often used in the treatment of systemic infections and methicillin-resistant Staphylococcus aureus (MRSA) in particular. Central nervous system side effects are usually mild and include chills and drug fever. There is no known increased risk of seizure. Educational Objective: Imipenem is associated with an increased risk of seizure.

Page 20: Miscellaneous 51

35% of people answered this question correctly; This subscription is licensed to user ID: 123489 only A 41-year-old executive presents to clinic complaining of generalized lethargy and weakness. He says he is "stressed out" at his job. Lately he finds himself drinking four to six cups of coffee per day just to stay focused on the tasks at hand. He claims to eat an otherwise nutritious diet and drinks a glass of wine before retiring to bed at night. Physical examination is normal. Upon further inquiry, he hesitantly reveals that he has frequently visited prostitutes while on business trips to foreign countries. An ELISA test for HIV antibodies returns with a positive result and is confirmed with the Western blot. The patient indicates an interest in beginning antiretroviral therapy. He insists, however, that his wife not be informed about his diagnosis because he suspects the news would compel her to seek a divorce. Which of the following is the most appropriate response to his request? A. Agree to maintain confidentiality regarding his HIV status B. Agree to maintain confidentiality regarding his HIV status, but then contact his wife to inform her once he leaves the office C. State that you must inform his wife about his HIV status because you are her primary care physician too D. Submit information regarding his HIV status to a state agency so that his wife will be contacted by an official third party E. Persuade him to tell his wife about his HIV status This subscription is licensed to user: roopika only User ID: 123489 Explanation: Patient confidentiality has been a sacred and fundamental principle of medicine since the writing of the Hippocratic Oath. In addressing an issue of such import as HIV status, then, it is extremely important to honor the patient's right to privacy and confidentiality to the greatest extent possible. However, most states also acknowledge the necessity of notifying individuals (such as the spouse in this case) who are known to be at risk for contracting HIV. The case of Tarasoff v. Regents of the University of California established that when a physician finds an identifiable third party to be foreseeably endangered due to a patient's conduct, that physician has a duty to warn the third party. Many medical ethicists argue that this legal precedent applies to the potential transmission of HIV as well. Therefore, the physician in this case should make every effort to convince the patient to inform his wife about his HIV status (Choice E), even if such a revelation results in divorce. To ease the patient's burden, the physician should also offer to help in conveying the difficult news to the patient's wife. If the patient refuses to inform his wife despite the physician's best efforts, most states allow for the physician to then convey the information to the at-risk spouse.

Page 21: Miscellaneous 51

Agreeing to maintain the patient's confidentiality (Choice A) puts the wife at great risk of contracting HIV and is therefore inappropriate. While it is ethical to ensure that the wife is informed about her risk of contracting HIV, a physician should never deceive his patient about his intentions (Choice B). The patient should be given sufficient opportunity to tell his wife about his HIV status before a third party intervenes (Choices C and D). Educational Objective: An HIV-positive patient should be persuaded to tell his spouse about his HIV status. If he refuses despite the physician's best efforts, most states allow for the physician to then convey the information to the at-risk spouse. 74% of people answered this question correctly; This subscription is licensed to user ID: 123489 only A 26-year-old Caucasian man comes to your office for a follow-up visit. One month ago, he came in for genetic counseling because he wanted to know if he had Huntington's disease. He currently has no complaints. H e does not have any medical problems. He denies the use of tobacco, alcohol, or drugs. His father was diagnosed with Huntington's disease at the age of 33 years, and died twelve years later, after experiencing a progressive and disabling neurologic deterioration. He has one older sister and one younger brother who are apparently healthy. He has been married for five years, and is the father of two children. His test result is out, and as you begin to tell him, he says, "I've changed my mind, doc. I don't want to know the test result." Which of the following is the most appropriate response to this patient? A. You should receive the test results. Remember, the future of your family is involved B. Come back when you are prepared to receive the results C. I cannot force you to receive the results, but they will be available if you change your mind D. Do not worry about the results; just forget about it and continue your life as if nothing has happened E. It is your responsibility to accept the results you have requested This subscription is licensed to user: roopika only User ID: 123489 Explanation: Huntington's disease is a non-treatable medical condition. This patient may be affected by anxiety, desperation, and even depression, if he finds out that he will develop the disease soon. Refusing the results could be his way of protecting himself from all these unpleasant emotions. Physicians must be aware of this kind of reasoning. In such a situation, the principle of "primun non nocere" (first do not harm) must be honored.

Page 22: Miscellaneous 51

Furthermore, the principle of autonomy includes the patient's right to be informed about his disease and all current available treatment, as well as the right to refuse any unwanted information. According to the ethical guidelines of the World Health Organization (WHO) and the UNESCO, "the wish of individuals and family not to know genetic information, including tests results, should be respected, except in testing of newborn babies or children for treatable conditions." (Choices A) It is unethical for a physician to use a paternalistic approach. A physician must respect his patient's decision. (Choice D) It is not advisable to give false reassurance. The disease in question can dramatically affect the patient's quality of life. (Choice B) It is incorrect to anticipate that the patient will acknowledge the results in the future. Furthermore, this statement may be interpreted by the patient as an act of nonchalance. A physician should always behave with empathy towards his patient. Educational Objective: The principle of autonomy includes the patient's right to be informed about his disease and all current available treatment, as well as the right to refuse any unwanted information. Physicians must honor this principle and respect their patients' wishes, especially when disclosing possible life-altering information to patients. 80% of people answered this question correctly; This subscription is licensed to user ID: 123489 only A 78-year-old Caucasian woman with a history of metastatic colon cancer, malignant ascites, and congestive heart failure collapses at her son's home. She is escorted by ambulance to the hospital, where she is admitted to the intensive care unit. In the days leading up to her admission she had complained of fever, productive cough, shortness of breath, and malaise. Shortly after admission she develops respiratory distress that requires sedation, intubation, and ventilatory support. Wide-spectrum intravenous antibiotics are begun to treat suspected pneumonia, and her diuretic dosage is increased. Responsible for her care are a number of specialists, including an oncologist, pulmonologist, cardiologist, and critical care hospitalist. Each physician meets separately with the patient's adult children every day to discuss his assessment and treatment recommendations. The patient's condition moderately improves but all attempts to wean her from the ventilator fail. As each day passes, the family members become increasingly irritable when discussing their mother's health problems, and finally accuse you of suggesting unnecessary procedures and treatments. Which of the following is most likely responsible for the family's negative attitude? A. Inadequate pain management B. Violation of patient's living will directive against intubation C. Poor nursing care D. Concern that health care insurance will not cover cost of hospital admission

Page 23: Miscellaneous 51

E. Lack of communication between physicians involved in patient care This subscription is licensed to user: roopika only User ID: 123489 Explanation: The issues of greatest concern for families of patients with cancer include pain management and the establishment of excellent communication with the health professionals involved. If multiple physicians are responsible for a patient's medical care, it is crucial that they communicate regularly about key treatment objectives and the preferred means of accomplishing those objectives. When such communication is present, all physicians involved are well equipped to provide family members with a clear and cohesive picture of the medical team's assessment and plan. If such communication is absent, however, family members are left feeling that care is being provided in a haphazard or ineffective fashion (Choice E). Since the patient is sedated and there is no reference made to her experiencing any pain or discomfort since her arrival at the hospital, it is unlikely that her family feels her pain management is inadequate (Choice A). No mention was made of a living will, and it is unlikely that the family members would express concern about intubation several days after it was performed (Choice B). There is no evidence that nursing care or the cost of hospital admission is of concern for the family members (Choices C and D). Educational Objective: If multiple physicians are responsible for a patient's medical care, it is crucial that they communicate regularly so that family members are provided with a clear and cohesive picture of the medical team's assessment and plan. 85% of people answered this question correctly; This subscription is licensed to user ID: 123489 only The following vignette applies to the next 3 items An 83-year-old Caucasian woman is brought to the office by her son for a routine follow-up visit. She was recently seen in the local emergency room for a fall. A CT scan of the head was normal. She was discharged to go home. Her other medical problems include Alzheimer's dementia, hypertension, diabetes mellitus - type 2, hearing loss, coronary artery disease, aortic stenosis, gastroesophageal reflux disease, transient ischemic attack, and osteoporosis. She does not use tobacco, alcohol, or drugs. She lives at home with her son. Her family history is not significant. Her medications include donepezil, glyburide, metformin, aspirin, metoprolol, ranitidine, and acetaminophen. Her temperature is 36.1 C (97 F), blood pressure is 160/90 mmHg on sitting and 130/70 on standing, pulse is 72/min, and respirations are 16/min. She does not recall the fall. She

Page 24: Miscellaneous 51

seems withdrawn and more quiet than at her last office visit three months ago. She lost around 7 lbs since the previous office visit. A brief examination shows dry mucus membranes, normal jugular venous pressure, clear lung fields, grade 2/6, and a systolic ejection murmur at the right second intercostal area. Item 1 of 3 Which of the following is the most appropriate next step in the management of this patient? A. Ask the son to leave the room. B. Review her medications with the son. C. Recommend nutritional supplements during meal times. D. Obtain a CT scan of the chest, abdomen, and pelvis to rule out malignancy. E. Start her on a selective serotonin blocker. This subscription is licensed to user: roopika only User ID: 123489 Explanation: The patient had an unexplained fall. She appears withdrawn and more quiet. She lost weight and appears dehydrated, which makes you wonder if the patient is not eating and drinking well. All these are very concerning and suggest elderly abuse. It is very important to interview such patients individually, and ask questions such as, "Is anyone hurting you?" (Choice B) Her medications should be reviewed, but this should be done only after abuse has been ruled out. (Choice C) Treatment options should be discussed after an appropriate evaluation of the patient. (Choice D) A work-up for malignancy may be indicated after a detailed patient interview and physical examination. (Choice E) The patient does not meet the criteria for depression at this point. Educational Objective: A high index of suspicion for abuse is required in the evaluation of elderly patients. The patient must be interviewed privately without the presence of family members or caregivers. 48% of people answered this question correctly; This subscription is licensed to user ID: 123489 only Item 2 of 3

Page 25: Miscellaneous 51

Further physical examination shows a MMSE score of 24. The patient is weak, and she requires assistance when changing/transferring positions or walking. She has some insight regarding her situation. Stat labs obtained in the office show a blood sugar of 260 mg/dL, BUN of 40mg/dL, serum creatinine of 1.7mg/dL, and hemoglobin of 10.2 g/dL. What would be your next best step? A. Admit the patient and evaluate further. B. Offer outpatient evaluation by an oncologist. C. Add insulin for her uncontrolled diabetes. D. Setup a home health nurse aid. E. Send her to nursing home. This subscription is licensed to user: roopika only User ID: 123489 Explanation: All patients who are in an abusive relationship should be separated from the abuser as soon as possible. This patient also appears dehydrated; she will require intravenous fluids and some physical therapy for her weakness. Her diabetes also needs to be controlled with an appropriate regimen. Adult Protection Service (APS) should be informed about this patient. Educational Objective: All patients who are in an abusive relationship should be separated from the abuser as soon as possible. 69% of people answered this question correctly; This subscription is licensed to user ID: 123489 only Item 3 of 3 The patient says that she is too old, and she does not want to have any evaluation. She also says that she is taking enough medications, and does not want to take any more. She wants to go home, and refuses nursing home placement. Two weeks later, you receive a call from a home health care nurse, who is concerned about the patient. She now lives alone and refuses home health services. What is the next best step? A. Respect her wishes and assure her that you will continue to follow. B. Tell her that she cannot stay at home alone since she is at risk for falls. C. Ask the patient why she is not interested in home health care. D. Tell her that you cannot continue to participate in her health care if she acts like this. E. Transfer her care to another physician.

Page 26: Miscellaneous 51

This subscription is licensed to user: roopika only User ID: 123489 Explanation: Legally, all mentally competent patients have the right to reside in an abusive relationship and refuse treatment. Although the physician has to respect this right, it remains his/her duty to identify any possible barriers to the appropriate health care of his/her patient. The physician must therefore attempt to find out why the patient is not interested in getting any help, and discuss possible options to resolve such issues. 76% of people answered this question correctly; This subscription is licensed to user ID: 123489 only A 78-year-old Caucasian man with end stage esophageal cancer is admitted to the hospital with severe malnutrition and failure to thrive. The patient's caretaker tells you that he has not been able to eat or drink for the last six weeks. His weight dropped from 160 pounds to 120 pounds during that time. The cancer has spread to his lungs and liver. The patient expresses his wishes to not receive any further treatment for the cancer, and specifies that he does not want any heroic measures or interventions done to keep him alive. Keeping his current clinical condition in mind, you think about offering hospice care to the patient. When is the most appropriate time to refer this patient to hospice care? A. He should be referred to hospice care now. B. He should be referred to hospice care two weeks prior to his anticipated death. C. He should be referred to hospice care two months before his anticipated death. D. He should be referred to hospice care after he has cleared his hospital bills. This subscription is licensed to user: roopika only User ID: 123489 Explanation: Hospice care is usually provided to terminally ill patients who have a predicted life expectancy of six months or less. The largest population of patients receiving hospice care consists of cancer patients. Other patients receiving hospice care have terminal medical conditions such as endstage cardiomyopathy, endstage chronic obstructive pulmonary disease, or pulmonary fibrosis. Hospice care is based on the principle of providing compassionate and comprehensive support and care, which includes, but is not limited to psychological, social, nursing, and palliative medical care to a dying patient. It also provides support and respite care to the family members or caregivers of the terminally ill patient. Hospice care is provided by a multidisciplinary team, which includes a registered nurse, nurse's aide, social workers, chaplains, and a hospice physician who closely coordinates with the patient's attending physician. It is usually provided at the patient's own home (home hospice care), but it can

Page 27: Miscellaneous 51

also be given as an inpatient hospice care for patients who are not functionally independent. (Choices B and C) There is a general tendency to delay hospice care to eligible patients by their caregivers or attending physicians. This leads to a shorter length of stay in hospice programs, and deprives the patients of the full benefits available to them. The patient in the above scenario has advanced esophageal cancer with an expected survival of less than six months. He should be referred to hospice care program now, or as soon as possible. Educational Objective: Patients with advanced metastatic cancers or other terminal illnesses with an expected life expectancy of less than six months should be evaluated for hospice care. 94% of people answered this question correctly; This subscription is licensed to user ID: 123489 only A 35-year-old white woman was admitted to the hospital. She is 38-weeks pregnant, and apparently had premature rupture of membranes at home. The delivery is uneventful; however, hours later, the newborn looks ill, sleepy, and has a poor appetite. Extensive work-up is completed, and a diagnosis of neonatal sepsis is made. The patient is concerned because she will be discharged tomorrow, but her baby needs to stay for at least a couple of days or more, in order to receive parenteral antibiotic therapy. Her insurance only covers admission for two days in the hospital for a non-complicated vaginal delivery. She requests you to "do something about it" so she can stay with her baby until he is ready for discharge. Which of the following is the most appropriate course of action? A. Don't worry, we will take care of that. I'll call the insurance company. They'll understand. B. Forget about the insurance. There will be no problem. C. I am sorry. Those are the insurance rules. We can try, but there is little hope. D. Be calm. I can write an order to admit you or lengthen your hospitalization E. Don't bother about it, your baby will be fine. You can go home and come back whenever you want. This subscription is licensed to user: roopika only User ID: 123489 Explanation: The physician must address the legitimate concern of the mother, explain the rules to her, and offer to help by attempting to talk to the insurance company about the rules on coverage; however, the physician must not make any guarantees to avoid any unrealistic expectations.

Page 28: Miscellaneous 51

(Choice D) Insurance rules are strict. If the physician tries to justify an extension of a hospitalization not based on clinical grounds, he can be accused of deceit or insurance fraud. (Choices A, B and E) It is not advisable to give any false reassurance or an optimistic superficial prognosis about the newborn's disease to make the mother feel better. Doing so may later create feelings of anger and deception in the mother. Educational Objective: The physician always has a commitment to tell the truth not only to the patient, but also to the insurance company that pays for his services. No false hope is to be created in the patient, and insurance fraud is to be avoided. In cases like this, the physician and patient may attempt to talk to the insurance company about the rules on coverage; however, the patient must be advised to avoid unrealistic expectations. 46% of people answered this question correctly; This subscription is licensed to user ID: 123489 only The following vignette applies to the next 2 items You are taking care of a 65-year-old male patient in the intensive care unit. The patient had an unwitnessed, out-of-hospital cardiac arrest two days ago. He was intubated and successfully resuscitated after a prolonged CPR in the field. Two days later, he remains intubated, and relies on full ventilator support. He is unresponsive to verbal and tactile stimuli. His temperature is 37.2 C (99 F), blood pressure is 110/70 mmHg, and pulse is 70/min. After a lengthy discussion with the patient's family, you all agree to have the patient's life-sustaining support withdrawn. Item 1 of 2 Which of the following will you document to determine that the patient meets the criteria for brain death? A. Absence of respiratory drive for 5 minutes off the ventilator B. Body temperature below 35 C C. EEG with nonspecific waveforms D. Intermittent cerebral circulation on cerebral doppler scanning E. Irreversible absence of cerebral and brainstem reflexes This subscription is licensed to user: roopika only User ID: 123489 Explanation: Brain death is defined as the cessation of cerebral and brain stem function. A person is considered legally dead in the United States when criteria for brain death have been demonstrated. One of the criteria to determine brain death is the irreversible absence of

Page 29: Miscellaneous 51

cerebral and brainstem reflexes including pupillary, oculocephalic, oculovestibular (caloric), corneal, gag, sucking, swallowing, and extensor posturing. Some of the other criteria for determination of brain death include: 1. Absence of respiratory drive (apnea) off the ventilator for a duration that is sufficient to produce hypercarbic drive (usually 10 to 20 minutes to achieve pCO2 of 50 to 60 mmHg) (Choice A). 2. Body temperature above 34 C (93.2 F) (Choice B). 3. EEG isoelectric for 30 minutes at maximal gain (Choice C). 4. Absence of cerebral circulation by Doppler or magnetic resonance angiography (Choice D). 5. At least 24 hours of observation in adults with anoxic-ischemic brain damage with a negative drug screen Educational Objective: Brain death is the irreversible absence of all cerebral and brainstem reflexes. There are no spontaneous breaths regardless of hypercarbia or hypoxemia. 71% of people answered this question correctly; This subscription is licensed to user ID: 123489 only Item 2 of 2 Using the above criteria, you are able to document that the patient is brain dead. You discuss these findings with his family and recommend that mechanical ventilator support be withdrawn. Everyone in the family appears to understand the situation and agrees to proceed with the withdrawal of ventilatory support. One relative then walks into the patient's room and notices some movements in the patient's legs. He gets angry with you and claims that the patient is not dead. Which of the following is the most appropriate response to the relative's reaction at this time? A. Tell the family that leg movements are inconsistent with the diagnosis of brain death B. Arrange for further testing to confirm your diagnosis C. Obtain a neurology consultation D. Tell the family that the relative is probably hallucinating E. Explain calmly that limb movements can be seen normally even in a brain dead person This subscription is licensed to user: roopika only User ID: 123489 Explanation: Brain death is defined as the irreversible absence of cerebral and brainstem reflexes including pupillary, oculocephalic, oculovestibular (caloric), corneal, gag, sucking, swallowing, and extensor posturing. Purely spinal reflexes, including tendon reflexes, plantar reflexes, and limb movements to painful stimuli can be present in these patients.

Page 30: Miscellaneous 51

Explaining this in a calm manner should be the appropriate response to dissipate the anger and anxiety among the family members. (Choice A) The presence of limb movements is not inconsistent with the diagnosis of brain death . (Choices B and C) Any further testing or consultations based on limb movements alone is not indicated if the criteria for brain death have been fulfilled earlier. Educational Objective: It is not unusual to see purely spinal reflexes manifesting as isolated limb movements in a brain dead person. 87% of people answered this question correctly; This subscription is licensed to user ID: 123489 only A 70-year-old Caucasian woman is brought to the office by her friend. She complains of extreme weakness, malaise and asthenia. She has had these symptoms for the past ten months, but they are getting progressively worse. She was diagnosed of agnogenic myeloid metaplasia two years ago. She was not offered bone marrow transplantation because of her age; instead, she received blood transfusions in more than ten occasions, as well as erythropoietin and androgens, which yielded poor results. She has no other medical problems and takes no other medication. She lives with her older son. Her temperature is 36.7 C (98 F), blood pressure is 120/70 mm Hg, pulse is 110/min and respirations are 20/min. She appears pale and undernourished, with significant muscular wasting. Auscultation reveals tachycardia; some rales are heard in both bases of the lung. Mild jugular venous distention and bilateral leg edema are present. Hepatosplenomegaly is present. The patient's laboratory tests reveal: CBCHb 4.6 g/dL Ht 14% MCV 76fl Platelet count 60,000/cmm Leukocyte count 2,500/cmm Serum chemistrySerum Na 132 mEq/L Serum K 3.8 mEq/L BUN 22 mg/dL Serum creatinine 1.3 mg/dL Calcium 10.0 mg/dL Blood glucose 86 mg/dL Three months ago, when her hemoglobin level was 7g/dL and her Ht 22%, the patient stated that she did not want any more blood transfusions or treatment. She has not

Page 31: Miscellaneous 51

changed her mind, and continues to refuse any type of therapy. Which of the following is the most appropriate decision? A. Allow her to go home and give her palliative medication B. Refer her immediately to the psychiatrist C. Refer her immediately to the emergency department D. Convince her to accept at least two units of packed red blood cells E. Try to locate her closest relatives while treating her for heart failure This subscription is licensed to user: roopika only User ID: 123489 Explanation: The only effective medical treatment for agnogenic myeloid metaplasia (AMM) is allogenic hematopoietic cell transplantation (allo-HCT). Unfortunately, only patients under 60 years of age can be considered as candidates. The rest are only treated palliatively. The median survival time is thirteen months for patients who are older than 60 years, experience weight loss, have a hemoglobin level lower than 10g/dL, leukopenia (WBC < 4,000 /cmm) or leukocytosis (WBC > 30,000/cmm), platelet count under 150,000/cmm, more than 2% of blasts, or hepatomegaly. This patient has all, but two, of the aforementioned risk factors. She is also developing high output heart failure and severe anemia as consequences of the disease. Her prognosis is very poor, and she feels that her quality of life is not adequate. In such a situation, the physician must respect the patient's decision, and send her home with palliative medication. The principle of autonomy includes a patient's right to refuse treatment. Unless the court rules that this terminally ill patient is incompetent, she can continue to exercise her right to choose what she thinks is best for her. (Choices C and D) The patient cannot be treated against her wishes because doing so violates her right to autonomy. (Choice B) Patients may be referred to the psychiatrist if they present with psychotic or depressive features. However, this patient has no such features. (Choice E) Locating the relatives is unnecessary. The patient has already expressed her wishes, which should be acknowledged and respected by the physician. Educational Objective: Patients with untreatable diseases (i.e., myeloid metaplasia), a poor quality of life, and those who have already tried different therapies in the past may sometimes refuse to accept any kind of treatment. The principle of autonomy includes a patient's right to refuse treatment in such situations. The physician must respect his patient's decision, and not look for extraordinary therapeutic measures, unless the patient's evaluation of his/her case is not realistic. Psychiatrists can be consulted if there is any suspicion of psychosis or depression in the patient.

Page 32: Miscellaneous 51

71% of people answered this question correctly; This subscription is licensed to user ID: 123489 only A 46-year-old man is admitted to the hospital with complaints of constant mid-abdominal pain for the past five days. He has a history of intravenous drug abuse, alcoholism, and chronic pancreatitis. He continues to drink heavily. He has had three recent admissions for pancreatitis in the past four months. You consult a gastroenterologist, a close friend of yours, for a possible endoscopic retrograde cholangiopancreatography. He has also known the patient from his previous admissions. After the procedure, he calls and tells you, "I do not want to get involved in this patient's care. He is extremely non-compliant and drug-seeking, and has slapped one of the nurses in the endoscopy lab for not giving him enough pain medications." You have been the patient's primary care provider for the past 12 years and know that the patient can be difficult at times. Which of the following is the most appropriate next step in the management of this patient? A. Ask him to withdraw from the patient's case if he wishes. B. Tell him that he can withdraw from the case but he will not be consulted again for any of your patients in the future. C. Ask him to document the patient's behavior in the chart and then withdraw from patient's case. D. Tell him that he cannot withdraw from the case once he is involved in patient care. E. Tell him that he can withdraw from the case if some other gastroenterologist is willing to take care of this patient. This subscription is licensed to user: roopika only User ID: 123489 Explanation: All physicians have a moral obligation to provide continuity of care for their patients. It is not appropriate to withdraw or discontinue patient care in the middle of a hospitalization or during an acute ongoing medical treatment. All physicians have the option to choose who they want to serve; however, once they are involved in a case, they should not neglect the patient. If a physician wishes to withdraw from a case, he can do so if he provides the patient or the caregivers a notice long in advance so as to sufficiently permit the transfer of medical care to another physician. In the above vignette, the gastroenterologist is already involved with the patient's care since he has already performed the procedure. He can opt to withdraw from the patient's case only if there is another gastroenterologist willing to assume patient care responsibility. (Choices A and B) Asking the physician to withdraw without securing another gastroenterologist for the patient is not appropriate. (Choice C) The patient's behavior should certainly be documented in the chart; however, this does not provide sufficient grounds for withdrawal in the middle of the treatment.

Page 33: Miscellaneous 51

(Choice D) As described above, the physician can withdraw from the case if another gastroenterologist is willing to assume the patient care responsibility. Educational Objective: Physicians can terminate the physician-patient relationship by providing a notice long in advance to sufficiently permit the transfer of care to another healthcare provider. 35% of people answered this question correctly; This subscription is licensed to user ID: 123489 only The following vignette applies to the next 2 items A 72-year-old Caucasian man is brought to the emergency department (ED) due to a sudden onset of severe difficulty in breathing. He has chronic obstructive pulmonary disease, and is on 2 liters of oxygen at all times. While in the ED, he develops progressive hypoxic respiratory failure, and is intubated by the ED physician. His son comes to the hospital an hour later, and gets very upset. He pulls you to one side of the room and says, "Please remove the tube. He never wanted to live like this." The patient does not have any advance directives for his health care. He regains consciousness for a few seconds while you were having a discussion with his son, but he was unable to state his wishes regarding life-sustaining treatment. Item 1 of 2 Which of the following is the most appropriate next step in the management of this patient? A. Tell his son that you cannot discontinue mechanical ventilation without an advanced directive. B. Ask his son to consult the rest of his family first. C. Ask him to explain the reason behind his statement in greater detail. D. Explain the consequences to the son and proceed with discontinuation of mechanical ventilation. E. Get the hospital ethics committee involved in the case. This subscription is licensed to user: roopika only User ID: 123489 Explanation: Physicians are often faced with difficult scenarios involving the withdrawal of life-sustaining treatment. The situation may arise when a patient is rendered incompetent or is unable to participate in decision-making, and does not have any advance directives. In such situations, the physician must still recognize and respect the patient's autonomy and right to make healthcare decisions. It is the physician's responsibility to act in the patient's best interest by identifying a surrogate who must make healthcare decisions for the patient based on substituted judgment. The surrogate speaks on the patient's behalf, and

Page 34: Miscellaneous 51

must have the most knowledge on what the patient would have done or wanted if he were able to make his own healthcare decisions. The patient's spouse or next of kin usually acts as the surrogate decision-maker in the absence of a formally or legally designated surrogate. The patient's son appears to have some insight into his father's wishes when he claimed that "he would have never wanted to live like this." The patient may have previously expressed his wishes regarding life-sustaining treatment to his son. The son should therefore be asked to provide more information and reasoning behind the decision to withdraw mechanical ventilation at this point. (Choice A) Telling the son that mechanical ventilation cannot be discontinued without an advance directive is incorrect. The son can assume the role of surrogate decision-maker as long as the physician believes that he is acting in the patient's best interest. (Choice B) The physician should discuss the situation with the son in greater detail before involving the family. (Choice D) Understanding the reason and thoughts behind the son's opinion should be attempted to ascertain if he can act as the patient's surrogate decision maker. Once a surrogate is identified, the decision to withdraw or maintain ventilatory support can then be made. (Choice E) Involvement of the hospital ethics committee is not required at this point. Educational Objective: Before making any decisions regarding the withdrawal of life support measures, it is important for a physician to act in the patient's best interest by identifying a surrogate, with whom he must effectively communicate and discuss all issues and concerns. 51% of people answered this question correctly; This subscription is licensed to user ID: 123489 only Item 2 of 2 The patient's whole family shows up in the hospital two hours later. He has two sons and three daughters who appear extremely concerned by the recent events. They all live in the same town and are very close to their parents. The patient lives alone. After prolonged discussions, the two sons agree and wish to proceed with the withdrawal of mechanical ventilation. The daughters are adamant about continuing life-sustaining treatment. The daughters tell you, "Our dad never said that he didn't want to live". Which of the following is the most appropriate next step in the management of this patient? A. Discontinue mechanical ventilation when the daughters are away. B. Tell the family that you will continue the mechanical ventilation as you do not want to act against some of the family members' wishes.

Page 35: Miscellaneous 51

C. Discontinue mechanical ventilation per the son's original request. D. Involve the hospital's ethics committee in decision-making and mediation process. This subscription is licensed to user: roopika only User ID: 123489 Explanation: Quite frequently, a physician is faced with a situation wherein multiple first-degree relatives cannot agree on the approach to medical care, despite appropriate and adequate counseling. In such cases of conflict, the hospital's ethics committee should be involved to act as a mediator between the different family members. In extreme cases, the case may need to be taken to court, where a guardian is appointed to assist in the medical decision-making. (Choices A, B, and C) All the family members should be involved in the decision-making process regarding the withdrawal of mechanical ventilation. Educational Objective: The hospital ethics committee should be involved if a conflict exists between multiple family members regarding the appropriate approach to the patient's medical care despite adequate mediation by the physician. 78% of people answered this question correctly; This subscription is licensed to user ID: 123489 only A 32-year-old pregnant woman who recently migrated from Eastern Asia is brought to the hospital by her husband. She is on her 37th week of gestation. She has been having regular uterine contractions for the last four hours. Examination reveals that she is in active labor. She agrees to be admitted for delivery. You then explain to the patient and her husband the various possible methods of delivery and the complications that can arise from them. The couple appears to understand everything that you explain. When you ask the patient to sign the consent sheet for treatment, she becomes reluctant, and tells you that women in their culture are not allowed to sign any papers if their husbands are alive. She asks you to have her husband sign the consent sheet for her treatment. Which of the following is the most appropriate reaction to the patient's request? A. Tell the patient that you would not be able to treat her if she refuses to sign the consent. B. Ask the hospital not to admit the patient if she refuses to sign the consent. C. Tell her that you would not be responsible for any malpractice claims if her husband signs the paper. D. Obtain a witness to the above conversation, and treat the patient. This subscription is licensed to user: roopika only User ID: 123489 Explanation:

Page 36: Miscellaneous 51

Informed consent involves a process whereby a patient or his/her surrogate decision-maker makes a healthcare decision based on the physician's recommendation. Before arriving at a decision, the patient and the healthcare professional work must together as a team. It is the physician's obligation to provide all the medical facts accurately to the patient or the surrogate decision maker in accordance with good medical practice. It is important to ensure that the information provided to the patient is adequate, and is easily and completely comprehended. The patient or surrogate then exercises the right of autonomy and makes the healthcare decision. In the above vignette, the patient was provided with adequate information. She and her husband appear to understand the information completely. She has a right to choose a surrogate decision maker, such as her husband, to make healthcare decisions on her behalf. Both the patient and her husband want to proceed with the planned treatment as well. The patient should be treated appropriately even if she refuses to sign the consent for treatment herself. (Choices A and B) Refusal of admission or treatment of the patient is unethical and inappropriate. (Choice C) This statement is not true. The physician has to assume responsibility for all actions which directly involves the management of his patient. Educational Objective: Informed consent is considered effective when sufficient information is provided by a physician, and is easily and adequately comprehended by the patient or their surrogate decision maker. 84% of people answered this question correctly; This subscription is licensed to user ID: 123489 only An 80-year-old Caucasian man is hospitalized because of ischemic stroke. He has a history of hypertension, chronic renal failure, chronic obstructive pulmonary disease (COPD), and degenerative joint disease. He has smoked one-and-a-half packs of cigarettes daily for 50 years. He lives at home, alone, on the third floor of a five-story building. After three days in the hospital, he is ready to be discharged home, but he has been left with severe neurologic sequelae. He cannot eat, walk, or move without assistance. His nearest relative is a second-generation nephew who states that he has no time nor resources to take care of him. Where should this patient be referred? A. To a nursing home B. To a skilled visiting nurse C. Home, with 24-hours Home Health Aid (HHA) service D. To a hospice This subscription is licensed to user: roopika only User ID: 123489 Explanation:

Page 37: Miscellaneous 51

The patient has multiple comorbidities and needs special and close care, which can only be offered by a nursing home. Even if the patient were declared mentally competent and refuses nursing home placement, the Social Services Administration and Adult Protection Services can refuse to allow him to return to his home unless there is somebody besides a home health aid (HHA) that can be responsible for his care. (Choice B) A visiting nurse with adequate skills can supervise the medication and health status, but cannot stay with the patient permanently. (Choice C) The patient cannot be taken care by a HHA alone, because HHA responsibilities do not include medication and periodical health evaluation. (Choice D) The patient does not have a terminal condition. He is not a candidate for hospice. Educational Objective: Elderly patients who are unable to perform the basic daily living activities and take care of themselves can return home only if they are under the care or responsibility of another person, friend, or relative, who will guarantee adherence to the treatment and medical follow-up. If this is not possible, and if the patient has multiple or serious medical conditions, he should be placed in a nursing home. 74% of people answered this question correctly; This subscription is licensed to user ID: 123489 only A 55-year-old man is brought to the emergency room by his neighbor for evaluation of crushing, substernal chest pain. The pain started two hours ago, and is accompanied by nausea and diaphoresis. The patient has a history of hypertension, diabetes mellitus and hyperlipidemia. His blood pressure is 124/72 mmHg and heart rate is 78/minute. An electrocardiogram done in the emergency room reveals the presence of 3 mm ST segment elevation in leads II, III and aVF. You immediately provide the initial care for an acute myocardial infarction, and call the cardiac catheterization team for an emergent coronary angiogram. When you return to the patient's room to reevaluate him, you find him standing next to his bed. He has taken off the electrodes and the cardiac monitor, and is getting dressed. Upon asking him, he tells you "I wish to go back home. My pain is much better now. I know that I am having a heart attack and I can die from it, but I do not want to undergo cardiac catheterization." Which of the following is the most appropriate next step in the management of this patient? A. Discharge the patient home and prescribe medications. B. Call security and restrain him. C. Call the neighbor for consent to treat the patient. D. Call the hospital ethics committee. E. Obtain permission through the hospital lawyer to treat the patient.

Page 38: Miscellaneous 51

This subscription is licensed to user: roopika only User ID: 123489 Explanation: When faced with a situation such as the one given above, the physician must always assess the patient's decision-making capacity by determining if the patient has the cognitive ability and emotional stability to make decisions about his healthcare. The patient should be able to understand and rationally evaluate the diagnosis, planned treatment, available alternative treatment, and risks of refusing treatment. Failure to recognize the lack or absence of decision-making capacity puts the patient at risk of harm if he refuses treatment. On the other hand, inappropriately coercing a patient into treatment deprives him of his fundamental right of autonomy and the right to make his own health care decisions. The patient in the above vignette appears to have a clear understanding of his medical condition and the risks (risk of death) of refusing treatment. He does not wish to undergo cardiac catheterization, and wants to go home. The best way to deal with this situation is to educate him regarding the risks and benefits of cardiac catheterization and alternative therapy. He should be questioned regarding the reason behind his decision. If he still does not want any treatment, then the physician has to discharge him with medications (and of course, document everything extensively). (Choice B) If there are any indications that other medical or psychological conditions may be impairing the patient's decision-making capacity, then he can be restrained and treated for these emergent conditions. (Choice C) The patient appears competent and has clearly expressed his wishes to the physician. There is no reason to contact the neighbor for decision-making or consent for procedures. (Choices D and E) The hospital ethics committee should be involved only if there are any doubts regarding the patient's decision-making capacity. Hospital attorneys are usually needed to appoint a surrogate decision-maker if the patient is deemed incompetent. These approaches can only be used in non-emergent situations. Educational Objective: Every adult patient with a sound mind and understanding has a right to make his own healthcare decisions. 88% of people answered this question correctly; This subscription is licensed to user ID: 123489 only A healthy, 26-year-old Caucasian woman comes to the clinic with her husband for a routine health maintenance examination. You are her primary care physician. She is currently pregnant and is also being regularly seen by a hematologist because of a probable Rh sensitization. She is Rh negative, and her husband is Rh positive. Her Coomb's test is currently positive. She discloses to you that she has been pregnant in the

Page 39: Miscellaneous 51

past from another man and she had an abortion. Her husband does not know this, and she does not want it to be revealed to him. A friend of her husband happens to be a physician, and he got some medical information about cases similar to his wife's condition. Her husband asks you how this is possible considering that this is their first child. Which of the following is the best answer? A. Tell him that he may need to talk to his wife about this. B. Explain to him that sensitization can sometimes occur during the first pregnancy due to unclear reasons, and this is currently a matter of research. C. Tell him that this happened because his wife had conceived in the past. D. Refer him to the hematologist for a better explanation. E. Tell him that more tests need to be done to address his question. This subscription is licensed to user: roopika only User ID: 123489 Explanation: Patient confidentiality is paramount in medical practice and has recently been enforced by the new HIPAA (Health Insurance Portability and Accountability Act) regulations. In this case, the best thing to do is to tell the husband to talk to his wife about that particular topic. (Choice C) If the husband's health or safety is not jeopardized by his wife's condition, the physician has no obligation to disclose any confidential information about her to him. (Choices B, D and E) The physician must not give false information or refer the patient to another physician to address this delicate topic, as he or she is the primary care physician. Educational Objective: A patient's confidentiality is paramount in medical practice. It should be honored in most circumstances, unless a situation jeopardizes the third party's (spouse or relative) health or safety (e.g. homicidal intentions, HIV, or STD infection). If these cases occur, and the patient is mentally competent, the physician should encourage her to disclose the truth to her partner before he/she is obliged to do it him/herself. 63% of people answered this question correctly; This subscription is licensed to user ID: 123489 only A 35-year-old man is brought to the emergency room after sustaining gunshots to the head. After initial hemodynamic stabilization, the patient is transferred to intensive care unit. The following day, after consultations with the neurosurgeons, you conclude that he is brain dead, and there is no chance of any meaningful recovery. While searching for a contact address, the nurse finds an organ donation card in his pocket. You contact his family, and they confirm that he had always wanted to donate his organs. Which of the following is the most appropriate next step in the management of this patient?

Page 40: Miscellaneous 51

A. Completely stop the mechanical ventilation. B. Discontinue intravenous hydration but continue ventilation. C. Decrease the ventilator rate to 4 breaths per minute. D. Have a surgical team on standby while waiting for cardiac arrest. E. Arrange for his transfer to a transplant center. This subscription is licensed to user: roopika only User ID: 123489 Explanation: Organ transplantation from cadaveric donors has become increasingly common in the past few years. Most of the organs are obtained from brain dead donors. The successful recovery of viable organs for transplantation depends on appropriate identification and medical care of brain dead patients. It also depends on the speed of recovery of the organs from the potential donors, with a shorter time interval between brain death and organ recovery leading to better outcomes. Any delay in organ procurement leads to an increase in the number and severity of complications; therefore, this patient should be immediately transferred to a specialized transplant center. Brain dead organ donors should ideally be managed in an intensive care setting. The goal of intensive medical care is to achieve hemodynamic stability and maintain physiologic homeostasis to improve the viability of the organs. Particular attention should be paid to maintain a normotensive and euvolemic state. (Choices A and C) One of the goals is to provide optimal ventilator support to prevent hypoxia and hypercapnia. Completely discontinuing or changing the ventilator setting is not appropriate when the patient is hemodynamically stable. (Choice B) Hypotension is very common in brain dead patients. It is related to the loss of sympathetic tone, systemic infections, volume depletion secondary to losses and diabetes insipidus. Adequate volume resuscitation is therefore an important step in ensuring donor organ viability. Intravenous hydration should be continued to maintain euvolemia and an adequate urine output. (Choice D) Waiting for a cardiac arrest before procuring the organs is not the standard of care, and is not recommended. Educational Objective: Coordination of care between the different teams involved in organ procurement is a crucial step in the management of brain dead organ donors. 67% of people answered this question correctly; This subscription is licensed to user ID: 123489 only A 46-year-old Caucasian woman comes to your office for a routine follow up visit. She was previously diagnosed with depression and obesity. She has smoked one pack of cigarettes daily for the past 30 years. She drinks one ounce of alcohol daily, and takes

Page 41: Miscellaneous 51

fluoxetine. Examination shows no abnormalities. The patient tells you that she read an article in the Internet, which discussed a study involving a new Chinese herb that was very effective for depression. According to the article, eight patients took the herb for six weeks, and five were cured. She wants to try it, but she wants to hear your advice first. Which of the following is the most appropriate response to the patient's inquiry? A. Do not use it. This herb does not work B. Do not use it. The results may be false C. Do not use it. There is not enough evidence D. You must use only the medications I prescribed you. E. Use it, but at your own risk. I do not recommend it This subscription is licensed to user: roopika only User ID: 123489 Explanation: The physician is being asked about a new herbal medication, of which he has no information. Data from only a single study is available. Five of eight patients who agreed to take the treatment responded "well" to treatment. There are many aspects to this study which makes its data insufficient to be considered as scientific proof or evidence. The population used was small, so the possibilities of obtaining an effect simply by chance (random error) are high. Other information about the study is also lacking. How were the patients diagnosed? Were their cases of depression equally severe? What were the adverse effects of the herbal medication? What other medications were the patients taking concurrently? Were the patients randomized to therapy? Were the patients and the physicians blinded to therapy? How high was the placebo effect in these cases? Such questions should first be answered adequately before the physician accepts the data from the study as accurate and reliable. (Choice D) The end of paternalism in medicine has led to a new era, in which the physician has to honor his patient's right to autonomy. A doctor cannot simply tell his patient to use only his prescriptions. He has to state the reasons why he thinks the new drug or herbal medicine should not be used. (Choice E) It is the physician's duty to protect the patient from a possibility of drug or herbal toxicity. "Primun non nocere" (first, to avoid harm), as Galen said. Patients must be counseled about the perils of a new, untested drug, and not be left on their own. (Choices A and B) There is not enough information from this case to state that the new medication does not work or that the study has false results. Educational Objective: Physicians must be cautious when questioned about new therapies that have not been adequately tested. They must counsel their patients to wait until further and more solid information is available, to avoid undesirable side effects. Their approach must avoid paternalism and unjustified prejudice against or in favor of particular therapies.

Page 42: Miscellaneous 51

66% of people answered this question correctly; This subscription is licensed to user ID: 123489 only A 42-year-old Caucasian man comes to the office for a follow-up visit. His father died at 40 years of age from Huntington's disease. He is worried about the possibility of having a similar illness. You have ordered genetic testing and he is positive for Huntington's disease. He has no other complaints or medical problems. He asks you to conceal this information from his relatives. What is the most appropriate decision in this situation? A. Do not inform the patient's relatives about his diagnosis B. Call the patient's relatives and tell them the diagnosis C. Inform only the patient's closest relatives who are at risk of the disease D. Consult with the hospital's Ethics Committee E. Document the refusal of the patient and make him sign it This subscription is licensed to user: roopika only User ID: 123489 Explanation: Physicians must be aware of the implications of identifying a serious hereditary illness in their patients. For instance, Huntington's disease and Tay-Sachs disease are hereditary conditions with progressive neurologic compromise that ultimately lead to death. If any of the abovementioned diseases are identified in a patient, the risk of the patient's relatives for having the same disease is highly increased; therefore, counseling of patients who are about to undergo genetic testing must be integrated in their management. According to the American Medical Association (AMA), such patients should be pre-counseled (before genetic testing) on the need to disclose the results of certain genetic tests to their relatives. When a patient refuses to disclose the information despite having positive results, the physician must respect his patient's decision; however, he should obtain proper documentation (with the patient's signature) of the patient's intention to conceal his diagnosis. (Choice A) In addition to concealing the information from the patient's relatives, the physician must obtain legal documentation of the patient's decision (i.e., the patient must sign a document/statement in the chart, indicating his refusal to disclose the information). Several cases have been filed and won against physicians who did not have the proper documentation of the patient's decision to conceal the test results from his/her relatives. (Choices B and C) The confidentiality act states that private and genetic information must not be disclosed to third parties. Civil and criminal actions are currently being taken towards physicians who have violated this law. (Choice D) The ethics committee cannot force the patient to disclose the information to another individual, and does not have the right to reveal it. Educational Objective:

Page 43: Miscellaneous 51

When a patient is diagnosed with a serious hereditary illness (e.g., Huntington's disease, Tay-Sachs disease), the risk of the patient's relatives for having the same disease is highly increased. Patients should thus be pre-counseled on the need to disclose the results of certain genetic tests. If the patient refuses to disclose the information despite having positive results, the physician must respect his patient's decision; however, he should obtain proper documentation (with the patient's signature) of the patient's intention to conceal his diagnosis. 24% of people answered this question correctly; This subscription is licensed to user ID: 123489 only A 30-year-old Caucasian male presents to clinic for a pre-employment physical examination. He is friendly and talkative, and says that he is excited about starting his "dream job" as an auto mechanic. The man's only medication is risperidone. A thorough history and review of his past medical records provides no justification for the use of this medication. When asked why he was prescribed risperidone, the man shrugs and says, "I just take whatever the Doctor gives me." His physical examination is unremarkable. Which of the following is the best means of approaching this situation? A. Advise the patient to press charges against his former physician B. Call his former physician to clarify why the prescription was given C. Report his former physician to the state medical board D. Report his former physician to hospital administration E. Call his former physician to recommend enrolling in a CME course on how and when to prescribe risperidone This subscription is licensed to user: roopika only User ID: 123489 Explanation: On occasion, the review of a patient's medical records will suggest that diagnoses made or treatments implemented by other physicians were unwarranted or were otherwise inappropriate. Professionalism dictates that colleagues be given the benefit of the doubt when such circumstances arise. Before assuming that such a review is definitive evidence of poor judgment or, worse yet, malpractice, it is important to contact the former physician for clarification. It may turn out that the patient's medical records are incomplete or that the patient neglected to mention a key point in his medical history. (Choice A) Advising the patient to press charges against his former physician is premature. The matter should be further explored with the former physician before any conclusions are reached. (Choices C and D) Reporting the former physician to the state medical board or to the hospital administration is not appropriate given how little information is available at this time. The matter should be further explored with the former physician before any conclusions are reached.

Page 44: Miscellaneous 51

(Choice E) Informing the former physician that he should enroll in a CME course on antipsychotics is rude and patronizing, and therefore unprofessional. Such a comment would be inappropriate under any circumstance. Educational Objective: If another physician's diagnosis or treatment appears inappropriate, the matter should first be clarified further with that physician before making any assumptions or recommendations to the patient. 96% of people answered this question correctly; This subscription is licensed to user ID: 123489 only A 68-year-old Caucasian female who presented with fever, cough and shortness of breath is admitted to the hospital. She has been admitted twice in the past six months due to right upper lobe pneumonia. Chest x-ray reveals right upper lobe pneumonia. A bronchoscopy is done followed by a biopsy, which is positive for an adenocarcinoma of the right lung. The patient's daughter makes a phone call to the physician from her office and requests information about the test results. The daughter mentions that she is the designated health care proxy for her mother. What is the most appropriate step at this point? A. Refuse to give her the information B. Discuss with her briefly about her mother's illness and tell her that she can obtain more information if she personally visits the hospital C. Inform her about the test results D. Explain to her that it would be unethical to divulge any information without prior permission from the patient E. Tell her to speak to her mother about the results This subscription is licensed to user: roopika only User ID: 123489 Explanation: Being the health care proxy, the daughter has the right to receive information about her mother's illness; however, the Patient Privacy Act states that it is best to give out only limited information about a patient diagnosis during a telephonic conversation. The best response in this case would be to try to explain to her that in the patient's highest interests, it is more desirable to obtain the complete information by personally visiting the hospital. (Choice A) The daughter has the right to be informed about her mother's illness. The physician cannot refuse to give her the information. (Choice C) Although the daughter has the right to be informed over the telephone, it would still be better if the physician encourages her to visit the hospital to obtain the complete information.

Page 45: Miscellaneous 51

(Choice D) The patient's permission is not required since the daughter is the appointed health care proxy. Although one might reason that there is no evidence supporting the daughter's claim of being the health care proxy, this choice would still not be the best response. Educational Objective: In accordance with the Patient Privacy Act, patient-related discussions should be kept to a minimum during a telephonic conversation. 22% of people answered this question correctly; This subscription is licensed to user ID: 123489 only A 70-year-old widowed African-American woman is admitted late one night to the hospital for an acute exacerbation of her congestive heart failure. During morning rounds, the medical team responsible for this woman's care meets with her for the first time. The senior resident begins to ask the patient some questions about her medical history and addresses her by first name. What is the appropriate response on the part of the attending physician? A. Address the patient by first name as well B. Address the patient with the salutation 'Madam' followed by her surname C. Address the patient with the salutation 'Mrs.' followed by her surname D. Address the patient with the salutation 'Ms.' followed by her surname E. Reprimand the resident and send him from the room This subscription is licensed to user: roopika only User ID: 123489 Explanation: New patients should always be addressed as "Ms. Smith" or "Mr. Smith," as that properly demonstrates the respect due the patient (Choice D). Keep in mind as well that older patients or patients with a different cultural heritage (such as the woman in this question) are often very sensitive to being spoken to in a manner that they perceive as disrespectful. The doctor-patient relationship may needlessly suffer if the doctor speaks informally without direct permission to do so. There is some debate regarding whether a patient should ever be called by first name, even if they so request it. Specifically, some ethicists insist that the level of formality in address must be applied mutually; therefore, if a patient is called by her first name, then the physician too must be called by his first name. Ultimately, it is best to err on the side of formality rather than informality. Addressing the patient by the first name (Choice A) is presumptive and risks alienating the patient. The salutation of "Madam" (Choice B) is typically used for individuals of high standing (e.g., "Madam President" or "Madam Ambassador"). Using this form of address with all patients may be interpreted as sarcasm.

Page 46: Miscellaneous 51

The salutation of "Mrs." (Choice C) should be used if the patient has indicated it to be her preference. Keep in mind that some women do not want reference made to their marital status, which is why the salutation "Ms." is more often used. Openly reprimanding the resident (Choice E) is excessively harsh if the resident otherwise appeared to have good intentions in speaking with the patient. It also creates an awkward situation for the patient, who may feel obligated to defend the resident. If an attending wants to discuss the matter of proper salutation with the resident, it is best to do so outside of the patient's presence. Educational Objective: New patients should be addressed as "Ms. Smith" or "Mr. Smith" and not by first name. 56% of people answered this question correctly; This subscription is licensed to user ID: 123489 only A 62-year-old male comes to the office for a routine examination. He does not have any chronic conditions and is not taking any medications at this time. His vital signs are normal and physical examination is unremarkable. His BMI has remained at 28 kg/m2 for the past several years. He intends to lose weight over the next few months through a combination of improved nutrition and thirty-minute walks every evening. He requests guidance regarding his daily consumption of one glass of orange juice each morning and one banana each afternoon. He is prepared to make any necessary changes to his diet and asks if he should replace either of these with alternative fruits. Which of the following is the most appropriate recommendation? A. Continue consumption of both orange juice and banana B. Continue drinking orange juice but stop eating banana C. Continue eating banana but stop drinking orange juice D. Stop consumption of both orange juice and banana E. Replace all carbohydrates with protein This subscription is licensed to user: roopika only User ID: 123489 Explanation: As overweight and obesity become more prevalent in industrialized nations, interest in achieving and maintaining an appropriate weight has similarly flourished. Overweight is defined by a BMI from 25-29.9 kg/m2, and obesity is defined by a BMI above 30 kg/m2. This patient has a BMI of 28 and is therefore considered overweight. He should pursue weight loss through the combination of reduced caloric intake and increased caloric expenditure. A gradual weight loss of 1-2 pounds per week is preferable, which can be accomplished by a caloric deficit of 500-1000 calories per day. It is important to ensure that any weight loss plan includes a balanced diet with sufficient vitamins and minerals.

Page 47: Miscellaneous 51

For a healthy diet, USDA and Department of Health and Human Services (HHS) has developed a food pyramid. This pyramid consists of different varieties of food and number of servings one should consume per day. The base of the pyramid consists of grain groups including bread, cereal, rice, and pasta. A healthy person can consume up to five to eleven servings, depending on the calorie need. The second group in this food pyramid consists of fruits and vegetables. A person should consume two to four servings of any fruits or vegetables per day. Oil, fats, and sweets constitute the top of the pyramid, and should be used sparingly. Between the fruit and oil groups lies the meat, poultry and nut group, and only around two servings per day is recommended. Milk, yogurt, and cheese also are similar to the meat group and one should consume two to three servings in a healthy diet. Three glasses of skim milk will provide the daily required allowance for calcium and possibly vitamin D. With new findings concerning the low-carbohydrate diet, this pyramid is likely to change in the near future. (Choice A) Orange juice and bananas are both nutritious choices if used in moderation. One eight-ounce serving of orange juice is flavinoid-rich and contains 100 calories and potassium, folate, and thiamin, as well as more than 100% of the recommended daily allowance of vitamin C. One banana contains 100 calories and is an excellent source of vitamin C, vitamin B6, potassium, and dietary fiber. Therefore, the patient should be advised that he can continue consuming both orange juice and banana, as long as he accounts for these calories when planning his daily meals. (Choice E) Replacing all carbohydrates with protein is not advocated by even the strictest of low-carbohydrate diets. Educational Objective: Patients interested in achieving weight loss should limit calorie intake while increasing calorie expenditure. A nutritious food plan should include five to nine servings of colorful fruits or vegetables per day. 64% of people answered this question correctly; This subscription is licensed to user ID: 123489 only A 74-year-old woman presents to the the emergency department (ED) with worsening shortness of breath, orthopnea, and cough productive of pinkish sputum. She is well known to you from her previous multiple admissions to the hospital. Her past medical history is significant for diabetes mellitus, hypertension, hypercholesterolemia, coronary artery disease, myocardial infarction, congestive heart failure, osteoarthritis, glaucoma, and peptic ulcer disease. Her temperature is 37.2C(99F), blood pressure is 140/90 mm Hg, pulse is 108/min, and respirations are 22/min. Physical examination reveals the presence of a third heart sound, bibasilar crackles and bilateral pitting pedal edema. Chest x-ray reveals bilateral haziness. EKG reveals evidence of an old anterior wall myocardial infarction. Her recent transthoracic echocardiogram showed an ejection fraction of 30%. She is admitted to the hospital and treated for exacerbation of congestive heart failure. At the time of discharge, her son requests you to persuade his mother to stay with him and his wife. He has made several attempts to convince her to stay with them in the past, but

Page 48: Miscellaneous 51

she always declines and says that she prefers to live alone. What would be the most appropriate step at this point? A. Convince her that she will be happier staying with her son and daughter-in-law. B. Explain to her that given her multiple health problems, it would not be a good idea for her to stay by herself. C. Ask her if there is a particular reason for her refusal to stay with her son. D. Discuss with her the advantages versus the disadvantages of staying with her son. E. Ask her to consider the advantages versus the disadvantages of staying with her son. This subscription is licensed to user: roopika only User ID: 123489 Explanation: A physician's foremost duty is to act as the patient's advocate. It is unethical to simply comply with the son's request and not have a discussion with the patient, who seems capable of making sound decisions. If the patient's health is significantly poor, the physician must show his concern for her welfare by explaining her state of health and advising against living alone. (Choice A) It is unethical for a physician to make assumptions on what will make the patient happier. (Choices C, D ane E) At this point, the physician should discuss the patient's health and advise against her living alone. The patient's reaction to the physician's advice will determine the latter's next plan of action. Unless she volunteers the reasons as to why she refuses to stay with her son, the physician cannot explore her decision to live alone; however, an attempt can be made to explain the potential advantages and disadvantages of staying with someone, such as her son. Asking her to consider these advantages and disadvantages may be appropriate, but this should only be done after discussing her health issues. Educational Objective: A physician's foremost duty is to act as the patient's advocate. If the patient's health is significantly poor, the physician must show his concern for her welfare by explaining her state of health and advising against living alone. 19% of people answered this question correctly; This subscription is licensed to user ID: 123489 only A 42-year-old Caucasian man comes to the office for a follow-up visit. He was evaluated one week ago because of recurrent right upper quadrant abdominal pain, which usually occurs after eating fatty meals. He has no other medical problems. He does not use tobacco, alcohol or drugs. His family history is not significant. He does not have any medications. Physical examination shows no abnormalities. Abdominal ultrasound was

Page 49: Miscellaneous 51

performed after the previous visit, and this reveals the presence of two middle-sized gallstones in the patient's gallbladder, without compromise of the bile ducts and no intrahepatic lesions. He is advised to undergo laparoscopic cholecystectomy. He agrees to have the procedure, but states one condition: if something goes wrong with the surgery and he has cardiorespiratory arrest or fatal arrhythmia, he does not want to be resuscitated or receive electric shocks for cardioversion. Despite a prolonged discussion about his decision, he does not change his mind. He decides to write an advance directive before surgery. Which of the following is the most appropriate course of action? A. Postpone surgery, withdraw from the patient's care and refer him to another specialist B. Proceed with surgery, request the patient to sign a written agreement and continue his follow up C. Proceed with surgery, request the patient to sign advance directives and continue his follow up D. Refer the patient to a psychiatrist prior to surgery E. Postpone surgery and call the closest relative. Ask him/her to advice the patient and make him change his mind This subscription is licensed to user: roopika only User ID: 123489 Explanation: The patient's right to autonomy allows him to decide what treatment he should receive, regardless of his type of illness and its severity. On the other hand, the physician has the duty to offer care to his patient, to show respect for the patient's autonomy, to avoid any harm, and to maximize the benefits of therapy. Although it is usually unethical for a physician to abandon his patient, he is authorized under the Physician's Ethics Code to refuse to care for and refer the patient to another colleague when there is a conflict of values. Other cases with possible conflict of values are: abortion, insurance fraud, false claims, absurd advance directives, etc. In this case, there is conflict regarding the patient's refusal of basic cardiac life support. Since the physician and patient cannot agree on this issue, the former must consider withdrawing from the patient's care and referring the latter to another colleague. (Choice C) Because it is the physician's duty to protect his patient from harm, it is unethical and incorrect to proceed with this elective procedure, especially since there is the possibility of fatal harm due to the patient's refusal of basic life support. (Choice D) Requesting a psychiatric evaluation will not solve the problem, unless the patient is suffering from a psychotic or depressive episode. (Choice E) Confidentiality impedes the physician to disclose the patient's condition to another person, even though he/she is a near relative. Only the patient has the right to authorize such communications.

Page 50: Miscellaneous 51

(Choice B) The patient's decision to refuse basic life support is extremely serious. Surgery should not be allowed to proceed until the patient desists of his idea. Educational Objective: The physician has the duty to offer care to his patient, to show respect for the patient's autonomy, to avoid any harm, and to maximize the benefits of therapy. Although it is unethical for the physician to abandon his patient, he is authorized under the Physician's Ethics Code to refuse to care for and refer the patient to another colleague when there is a conflict of values. Other cases with possible conflict in values are: abortion, insurance fraud, false claims, absurd advance directives, etc. 5% of people answered this question correctly; This subscription is licensed to user ID: 123489 only A 55-year-old morbidly obese Caucasian woman comes to the office for a follow-up visit. She was recently diagnosed with diabetes mellitus type 2. Her other medical problems are hypertension and hyperlipidemia. She denies the use of tobacco, alcohol or drugs. Her medications include lisinopril, metformin, pravastatin, and hydrochlorothiazide. Examination shows no abnormalities. She currently weighs 224 lbs (101 Kg), which is a couple of pounds more than her previously recorded weight on her last visit. She admits to not following a diabetic diet because of her appetite. She also missed her previous appointment, which was scheduled two months ago. When asked if there were any reasons for her inadherence, she replied that she had gone to an alternative medicine specialist during the time that she was scheduled for a follow-up visit. Because she did not see any favorable results with the medications from the alternative specialist, she decided to stop seeing him. She has now started using a Chinese herb for obesity called ma huang . One of her friends tried it before and succeeded in losing weight. Despite counseling, the patient insists on taking ma huang and declares that she has lost faith in traditional medicine, as she has not been able to lose weight by any means. Which of the following are the most appropriate actions to be taken regarding this patient's decision? A. Allow her to continue taking ma huang, but continue her follow up. B. Respect the patient's decision, but refer her to an alternative medicine specialist. C. Obtain a written agreement in which the patient accepts to take the herb, in opposition to the physician's advice. At the same time, encourage her to return for follow up. D. Obtain a written agreement in which the patient accepts to take the herb, in opposition to the physician's advice. At the same time, refer her to another physician. E. Allow the patient to continue taking the herb, but document the clinical encounter carefully in her chart. This subscription is licensed to user: roopika only User ID: 123489 Explanation:

Page 51: Miscellaneous 51

Physicians must be cautious because some herbal medications that have been already taken out of the market continue to be sold under their botanical or equivalent name. For instance, ma huang is the Chinese name for an herbal medicine that can cause cardiac toxicity and has been reported as the cause of death in some cases. It is better known as ephedra. The patient in this case is therefore exposed to a dangerous medication. In the case of Schneider vs. Revici, a patient with breast cancer refused surgical treatment and sought alternative therapy. Because the patient had signed a consent form which released the physician from liability, the court ruled that the patient willingly accepted unconventional treatment and rejected the recommended standard medical methods; the patient was thus considered responsible for all consequences, and this effectively relieved the physician of a malpractice liability; however, simply obtaining a written agreement for the physician's protection should not be the only action taken. Reports from the Ethics Committee of the American Medical Association (AMA) state that the physician has the duty not to abandon the patient, unless there is a clear and insoluble conflict of values in the relationship. To correlate this with the above vignette, the physician should then advise the patient to engage in regular follow-up visits in order to identify and emergently treat any subsequent complications that may arise from use of the herbal medication; however, if the patient refuses, the physician must respect this decision. (Choice A) The patient is using a very dangerous medication. For this reason, she must be advised against the use of ma huang. (Choice B) Referral of the patient to an alternative medicine specialist would have been acceptable if the herbal supplement in question had beneficial effects and was considered non-dangerous. (Choice D) Careful documentation of the patient's use of a potentially toxic herbal medication despite the physician's advice is useful; however, this measure is not adequate to avoid malpractice claims. Furthermore, referring the patient to a colleague can provoke questions regarding moral conduct. Educational Objective: Physicians must be aware of the FDA-prohibited herbal medications that may be inadvertently obtained under different names. Examples are ephedra (Ma huang), licorice (Glycyrrhiza glabra), Panax pseudoginseng (San qi) or Saw palmetto (Serenoa repens). When the patient decides to take herbal medications against medical advice, the physician must obtain a written agreement, which releases him/her from any liability. In addition, the physician should attempt to follow the patient's health status and not abandon him/her, despite his/her decision. 38% of people answered this question correctly; This subscription is licensed to user ID: 123489 only

Page 52: Miscellaneous 51

A 27-year-old gravida 2, para 1 Caucasian woman at 38 weeks of gestation was admitted to the labor and delivery unit after the spontaneous rupture of membranes. After an unremarkable labor she vaginally delivered a son weighing 3720 grams (8 lbs, 3 oz). Shortly after birth, the infant was found to have hemolytic disease of the newborn secondary to Rh incompatibility. The infant's serum total bilirubin concentrations then rose rapidly and now, at 24 hours after delivery, are measured at 22.8 mg/dL. Intensive phototherapy has not adequately reduced the bilirubin concentration in the infant's blood. The parents of this child are Jehovah's Witness adherents and refuse the proposed exchange transfusion. Without the transfusion, the infant will likely suffer severe brain damage or death. What is the next best step in managing this boy's care? A. Administer RhoGAM to mother and infant B. Continue intensive phototherapy for additional 72 hours C. Proceed immediately with exchange transfusion to prevent kernicterus D. Consult with the hospital's ethics committee about seeking court injunction to mandate exchange transfusion E. Transfer infant to another hospital This subscription is licensed to user: roopika only User ID: 123489 Explanation: In difficult ethical situations such as this one, medical practitioners must seek to balance the autonomy of the family with the welfare of the child. Parental wishes should be honored within certain parameters. For instance, when surgeries are performed on Jehovah's Witness adherents (be they adults or children), specific blood-sparing techniques or erythropoietin may allow for excellent outcomes. However, in circumstances in which an infant's survival is at stake, it is appropriate for the medical team to seek approval from the courts to proceed with a blood transfusion (Choice D). Continuing the intensive phototherapy (Choice B) is inadvisable. Typically, phototherapy will result in a decrease in the bilirubin within 4-6 hours. Waiting another 72 hours puts the child at serious risk for developing kernicterus. In a R (D)-negative woman carrying a fetus that is or may be R (D)-positive, RhoGAM (anti-D immune globulin) must be given as a prophylactic measure at 28 weeks of gestation, after clinical procedures or trauma that can cause fetomaternal hemorrhage, and at the conclusion of any pregnancy. Once alloimmunization to the Rh (D) antigen has been established, however, RhoGAM administration (Choice A) is no longer effective. It therefore should not be administered to the mother in this case. Moreover, there is no current indication for the administration of RhoGAM to children, including those with hemolytic disease of the newborn. Proceeding with the exchange transfusion (Choice C) may save the infant's life, but invites litigation if done without first obtaining the approval of the court.

Page 53: Miscellaneous 51

Transferring the infant to another hospital (Choice E) irresponsibly evades the ethical conflict by placing the child's care in the hands of physicians who do not know his history. Transfers should only be initiated if the hospital currently caring for the boy is unable to properly address his medical issues, which was not implied in this case. Educational Objective: In providing medical care, clinicians must seek to balance the autonomy of the family with the welfare of the child. If the child's life is endangered, it is appropriate for the clinician to seek approval from the courts to proceed with treatments deemed medically necessary. 22% of people answered this question correctly; This subscription is licensed to user ID: 123489 only The following vignette applies to the next 2 items A 63-year-old Caucasian male presents to his primary care physician for a new patient visit. He is a pleasant and talkative man who has no new complaints; he reports that he "has never felt healthier." He has come prepared with a list of regularly taken medications. Included on the list are atenolol, sildenafil, terbinafine, minoxidil (topical), aspirin, and saw palmetto. He has no known drug allergies and has a remote history of cholecystectomy. He drinks one ounce of bourbon before bed each night. He has a remote 15-pack-year history of smoking cigarettes and has never used recreational drugs. He is married and has three grown children, and was recently transferred to this city to assume direction of a local factory. Physical examination is remarkable for a soft mid-systolic murmur that is loudest in the second right intercostal space. There is some bony enlargement of the distal interphalangeal joints bilaterally. The patient wishes to discuss his medication list in more depth and indicates an increasing interest in alternative medicines. Item 1 of 2 Which of the following conditions is saw palmetto most commonly used to treat? A. Anxiety B. Osteoarthritis C. Benign prostatic hyperplasia D. Hypercholesterolemia E. Depression This subscription is licensed to user: roopika only User ID: 123489 Explanation: Alternative medicine is enjoying an increasingly wide audience within the United States, with as many as 83 million adults reporting the usage of complementary therapies. Herbal

Page 54: Miscellaneous 51

preparations are not regulated by governmental agencies and are generally classified as food or dietary supplements, enabling manufacturers to avoid the scientific scrutiny exercised when prescription drugs are readied for market. The limited data that exists regarding herbal preparations is, unfortunately, often flawed. Despite these difficulties, health care practitioners should make an effort to recognize the primary indications and adverse effects of the more popular herbal remedies. Saw palmetto is one such herbal preparation, and men most often use it for the treatment of benign prostatic hyperplasia (Choice C). Saw palmetto has been shown to improve urinary symptom scores, nocturia, and peak urine flow. Its efficacy appears similar to that of finasteride, and adverse effects are unusual and mild. Anxiety (Choice A) and insomnia are sometimes treated with kava, an herbal preparation originally used in Polynesia. Multiple placebo-controlled trials demonstrated that kava has significant anxiolytic properties. Osteoarthritis (Choice B) is often treated with glucosamine and/or chondroitin, two preparations generally considered to be safe. Glucosamine may alter glucose regulation or sensitivity to insulin, and diabetic patients should be cautioned accordingly. Hypercholesterolemia (Choice D) is sometimes treated with preparations of garlic. There is limited data regarding its efficacy. St. John's wort is frequently used in the treatment of depression (Choice E). The available clinical data suggests that it is significantly more effective than placebo for most patients. In vitro, it appears to inhibit the reuptake of serotonin, dopamine, and norepinephrine. Educational Objective: Saw palmetto is a popular herbal preparation used in the treatment of benign prostatic hyperplasia. Kava is used for anxiety and insomnia. Garlic is used for hypercholesterolemia. St. John's wort is frequently used in the treatment of depression. Glucosamine is used for osteoarthritis. *Extremely important question for USMLE step-3 48% of people answered this question correctly; This subscription is licensed to user ID: 123489 only Item 2 of 2 The patient is counseled appropriately regarding his use of alternative medicines. Which of the following side effects is associated most strongly with saw palmetto? A. Bleeding and platelet dysfunction B. Hepatotoxicity C. Hypertension D. Nephrotoxicity

Page 55: Miscellaneous 51

E. Photosensitization This subscription is licensed to user: roopika only User ID: 123489 Explanation: The milder side effects observed with the use of saw palmetto are similar to those seen with placebo and include headache, nausea, and dizziness. Hypertension (Choice C) is the most serious adverse effect noted, and one study found it occurred in 3.1% of patients taking saw palmetto. Bleeding and platelet dysfunction (Choice A) are associated with usage of gingko biloba and garlic, two popular herbal preparations. Hepatotoxicity (Choice B) in the form of hepatitis, cirrhosis, and liver failure is associated with kava. Other herbal supplements known to cause liver toxicity include comfrey, borage leaf, coltsfoot, ephedra, chaparral, germander, and a Chinese medicine called jin bu huan. Nephrotoxicity (Choice D) in the form of fulminant tubulointerstitial renal disease has been observed with the use of some Chinese herbs, especially those containing artistolochic acid. This particular compound is included in many Chinese herbal weight loss regimens. The concurrent use of renal vasoconstrictors fenfluramine and diethylpropion may hasten the development of nephrotoxicity. The most common acute drug reactions to St. John's wort (hypericum) include gastrointestinal distress, dizziness, fatigue, and dry mouth. Longer-term studies have found an increase in anorgasmia, urinary frequency, and swelling. Photosensitization (Choice E) can be seen with intravenous administration of hypericum in HIV-positive patients. Educational Objective: Hypertension is a side effect of saw palmetto. Bleeding and platelet dysfunction can be seen with Ginkgo and garlic (check bleeding time). Hepatotoxicity is seen with Kava. 15% of people answered this question correctly; This subscription is licensed to user ID: 123489 only A 43-year-old male is brought to the emergency department after incurring injuries during a fight at a local bar. He is escorted by the police. He smells heavily of alcohol, is slurring his words, and has difficulty standing and walking. It is unclear if he experienced a loss of consciousness. Physical examination reveals numerous contusions and lacerations on his face that will require suturing. The soft tissue of his left orbit is swollen and erythematous and he appears unable to open his left eye. There is a freely bleeding

Page 56: Miscellaneous 51

five-centimeter scalp laceration in the right temporal region where the patient struck his head against a concrete ledge in the parking lot. When he is informed that he will need to have his lacerations sutured and imaging of his skull performed, he becomes very belligerent. He begins to shout and curse, claiming that the staff is harassing him and that he will sue the hospital for violation of his civil rights. He demands to be released immediately and says that he is "perfectly fine." What is the most appropriate response on the behalf of the physician? A. Administer disulfiram and admit the patient to the psychiatric ward B. Have the patient sign paperwork releasing the hospital of liability before discharging him C. Obtain a court order to treat the patient D. Obtain consent from patient's spouse to treat the patient E. Physically restrain and treat the patient This subscription is licensed to user: roopika only User ID: 123489 Explanation: When an intoxicated patient exhibits behavior that puts him in danger of harming himself or others, he should be treated under the doctrine of implied consent. This doctrine is based on the understanding that, were the patient competent and sober, he would wish to be treated. This patient should therefore be physically restrained with the help of the police, if necessary, and all appropriate evaluation and treatment performed (Choice E). Even though this patient's intoxication interferes with his ability to fully comprehend the magnitude of the situation, it is important to calmly explain the reasoning behind the restraint and the treatment plans that will follow thereafter. The administration of disulfiram would not be of help at this point, since it is a medication designed to dissuade individuals from drinking alcohol. It is not used when the patient is already intoxicated. Moreover, it is only appropriate to admit patients to the psychiatric ward when it appears they have psychiatric issues (Choice A). Since this patient is intoxicated and may have incurred a life-threatening injury, it is inappropriate to allow him to be discharged against medical advice - whether or not he has released the hospital from liability. Intoxicated patients are poorly equipped to make decisions about their need for medical care, and therefore should be treated under the guidelines of implied consent (Choice B). When an individual is at risk for harming himself or others, the courts expect hospital personnel to treat the patient on an emergent basis (Choice C). Since the patient is an adult whose wishes have already been made clear, it is not necessary to contact his spouse in an effort to obtain consent (Choice D).

Page 57: Miscellaneous 51

Educational Objective: Intoxicated patients who may have incurred life-threatening injuries should be physically restrained and treated. 60% of people answered this question correctly; This subscription is licensed to user ID: 123489 only A 37-year-old asymptomatic Caucasian man comes to the physician for a health maintenance evaluation. He has no complaints. He has diabetes mellitus type 2. He is a truck driver. He has smoked one pack of cigarettes daily for 20 years, but he quit 5 years ago. He eats a lot of junk food, and drinks 4-5 bottles of beer every other day. His family history is not significant. His daily medications consist of glyburide and metformin. His vital signs are within normal limits. Examination shows no abnormalities. Which of the following is the most effective strategy to deal with the patient's alcohol consumption? A. You must avoid drinking alcohol B. You can continue drinking, but in a moderate manner. Alcohol is good for your heart C. We must talk about your drinking and how it affects your driving D. It is not advisable for you to drink alcohol because you have diabetes E. It is not advisable for you to drink alcohol because you are on metformin This subscription is licensed to user: roopika only User ID: 123489 Explanation: Alcoholism can be very dangerous in type 2 diabetics. It especially increases the risk of hypoglycemia, which can be potentially hazardous to this patient and to other people since he is a truck driver. The patient's history of alcohol consumption therefore warrants further investigation. He must be screened for alcoholism, treated accordingly, and counseled on responsible drinking habits. (Choice A) The paternalistic approach to medicine is now considered unethical. Physicians must correctly explain the reasons behind a necessary change in the patient's behavior. Furthermore, total alcohol abstinence is not needed in this case, because the patient does not have a history of alcoholism in the family, pancreatic or liver disease, hypertriglyceridemia, heart failure or previous alcohol-related problems. (Choice B) Although moderate alcohol ingestion has proven to be beneficial for the heart through its effect on the elevation of HDL cholesterol and the action of the antioxidants found in red wine and some beers, it can also pose a risk to the patient's health, as it can produce chronic liver disease, cardiomyopathy and dependence. Before a physician promotes or advices against alcohol consumption, he must therefore first explore the situation further by obtaining a complete history.

Page 58: Miscellaneous 51

(Choice D) The American Diabetes Association recommends no more than two drinks per day for men and no more than one drink per day for women, provided that there is adequate management/control of the patient's diabetic status. (Choice E) Excessive alcohol intake and concomitant metformin use increase the risk of hypoglycemia and lactic acidosis; however, this fact is not an indication to avoid alcohol intake unless the patient has significant liver, cardiac or renal compromise, or is older than 65 years. 54% of people answered this question correctly; This subscription is licensed to user ID: 123489 only A 34-year-old Caucasian general contractor comes to the office and requests to be seen that same day even if he did not previously schedule an appointment. His past medical history is significant for disk atrophy and probable thecal sac entrapment at C2, C3, and C4. He has a three-year history of severe, shooting pain with extension of his neck. He also has a large skin graft on his left upper thigh from a hot water heater burn. Having recently moved into the area, he has not yet established a relationship with a new primary care physician and asks that you assume his care. His former physician provided him with high-dose opioids that enabled him to continue working. Physical examination reveals numbness and weakness of both arms. Six months ago, the numbness and weakness extended only to his shoulders. You recommend a neurosurgical consultation, and explain that surgical intervention may be necessary to prevent the progression of his symptoms. He refuses to be referred to a neurosurgeon, and he asks for a refill of his pain medication. You express great concern about the risks involved with taking this dosage of pain medication and with forgoing a neurosurgical consult given his condition, but he dismisses your comments. Which of the following is the most appropriate response? A. Here is the refill of your pain medication. See me in two weeks or as needed B. Here is a prescription for ketorolac. See me in two weeks or as needed C. Return to see me in two weeks when this refill of your pain medication runs out. You have until then to consider the neurosurgery consult D. I am referring you to a physical therapist. See me in two weeks or as needed E. I cannot refill your pain medication. You will have to see another physician This subscription is licensed to user: roopika only User ID: 123489 Explanation: Many physicians find the management of patients who require high doses of narcotics challenging. Most likely to arouse suspicion are those who insist upon narcotics during the first encounter with a physician. Nevertheless, if there is clinical evidence that the patient's symptoms are genuine, it is appropriate to provide a two-week refill of a narcotic prescription in an effort to provide continuity of care. In this case, it will also allow the

Page 59: Miscellaneous 51

patient further time to consider the recommended neurosurgical consultation and the invasive procedures that may be necessary to preserve his long-term function. (Choice A) Providing a refill of the narcotics without further discussion of the need for a neurosurgical consult does the patient a disservice because he may develop permanent, severe disability without relief of his spinal cord compression. It is the physician's obligation to continue to strongly recommend the neurosurgical consult and to remind the patient of the risks he faces. (Choice B) Prescribing an alternative medication such as ketorolac is inadvisable because it would likely provide inadequate pain relief and has the potential to cause significant adverse effects (e.g., gastrointestinal bleeding). In addition, since this patient has used narcotics for so long, he is likely physically dependent upon them and would suffer from withdrawal symptoms were he not appropriately tapered first. (Choice D) Although physical therapy may be indicated after neurosurgical intervention, it is unlikely to be of much help at this time and has the potential to exacerbate his condition. Moreover, the patient's need for pain relief is not addressed at all by this reply. (Choice E) Refusing to prescribe higher doses of narcotics as a general rule is overly limiting. Some patients do have a legitimate need for these medications, and should have their pain relieved when possible. Educational Objective: If there is clinical evidence that a patient is in pain, it is appropriate to provide a short-term narcotic prescription while the patient considers his treatment options. 50% of people answered this question correctly; This subscription is licensed to user ID: 123489 only A 76-year-old female is seen in the clinic with numerous medical complaints. She is scheduled to undergo elective surgery for right hip replacement. She says she is tired, has a very poor appetite, and does not sleep well at night. She denies any fevers, chills, chest pain, or shortness of breath. Her ECG and chest x-ray are normal. She cannot remember which medications she takes. What is the most important factor in perioperative adverse drug reaction in the elderly population? A. Age B. Multiple medications C. Reduced renal function D. Gastric alkalinity E. Reduced total body water This subscription is licensed to user: roopika only User ID: 123489 Explanation:

Page 60: Miscellaneous 51

Multiple medications are the most common factor in predisposing elderly patients to an increased incidence of adverse drug reactions. Patients over age 65 take an average of two to six prescribed drugs and one to three non-prescription drugs daily. These patients are at an increased risk preoperatively for anesthetic and operative drug reactions. The most common medications associated with adverse drug reactions include anti-psychotics, anti-hypertensives, sedatives, diuretics, NSAIDs, corticosteroids, and digoxin. (Choice D) Most elderly patients have an increased gastric pH, which tends to increase the absorption of drugs, thereby adding to the circulating levels of antagonistic compounds. (Choices C and E) The gradual reduction in the glomerular filtration rate in the elderly also adds to the circulating drug levels. This is further compounded by the fact that elderly patients have reduced levels of total body water. (Choice A) Age, itself, does not increase the risk of a perioperative adverse drug reaction. Educational Objective: Multiple medications are a major cause of adverse drug reactions in the elderly. 59% of people answered this question correctly; This subscription is licensed to user ID: 123489 only